Rocksolid Light

Welcome to novaBBS (click a section below)

mail  files  register  newsreader  groups  login

Message-ID:  

<liiwi> udp - universal dropping of an pigeon


tech / sci.math / Re: DC Proof is the biggest teaching mistake

SubjectAuthor
* DC Proof is the biggest teaching mistakeMostowski Collapse
+* Re: DC Proof is the biggest teaching mistakeDan Christensen
|`* Re: DC Proof is the biggest teaching mistakeMostowski Collapse
| +* Re: DC Proof is the biggest teaching mistakeMostowski Collapse
| |+* Re: DC Proof is the biggest teaching mistakeKip Foh
| ||`* Re: DC Proof is the biggest teaching mistakeMostowski Collapse
| || +* Re: DC Proof is the biggest teaching mistakeMichael Moroney
| || |+- Re: DC Proof is the biggest teaching mistakeKip Foh
| || |`* Re: DC Proof is the biggest teaching mistakeMostowski Collapse
| || | `- Re: DC Proof is the biggest teaching mistakeKip Foh
| || `- Re: DC Proof is the biggest teaching mistakeKip Foh
| |`* Re: DC Proof is the biggest teaching mistakeDan Christensen
| | `* Re: DC Proof is the biggest teaching mistakeMostowski Collapse
| |  `* Re: DC Proof is the biggest teaching mistakeDan Christensen
| |   `* Re: DC Proof is the biggest teaching mistakeMostowski Collapse
| |    `* Re: DC Proof is the biggest teaching mistakeMostowski Collapse
| |     +* Re: DC Proof is the biggest teaching mistakeMostowski Collapse
| |     |`* Re: DC Proof is the biggest teaching mistakeMostowski Collapse
| |     | `- Re: DC Proof is the biggest teaching mistakeMostowski Collapse
| |     `* Re: DC Proof is the biggest teaching mistakeDan Christensen
| |      `* Re: DC Proof is the biggest teaching mistakeMostowski Collapse
| |       `* Re: DC Proof is the biggest teaching mistakeMostowski Collapse
| |        `* Re: DC Proof is the biggest teaching mistakeMostowski Collapse
| |         `* Re: DC Proof is the biggest teaching mistakeMostowski Collapse
| |          `* Re: DC Proof is the biggest teaching mistakeMostowski Collapse
| |           `* Re: DC Proof is the biggest teaching mistakeMostowski Collapse
| |            `* Re: DC Proof is the biggest teaching mistakeMostowski Collapse
| |             +- Re: DC Proof is the biggest teaching mistakeMostowski Collapse
| |             +- Re: DC Proof is the biggest teaching mistakeMostowski Collapse
| |             +- Re: DC Proof is the biggest teaching mistakeMostowski Collapse
| |             +- Re: DC Proof is the biggest teaching mistakeMostowski Collapse
| |             +- Re: DC Proof is the biggest teaching mistakeDan Christensen
| |             +- Re: DC Proof is the biggest teaching mistakeMostowski Collapse
| |             +- Re: DC Proof is the biggest teaching mistakeMostowski Collapse
| |             +- Re: DC Proof is the biggest teaching mistakeDan Christensen
| |             +- Re: DC Proof is the biggest teaching mistakeMostowski Collapse
| |             +- Re: DC Proof is the biggest teaching mistakeMostowski Collapse
| |             +- Re: DC Proof is the biggest teaching mistakeMostowski Collapse
| |             +- Re: DC Proof is the biggest teaching mistakeMostowski Collapse
| |             +- Re: DC Proof is the biggest teaching mistakeDan Christensen
| |             +- Re: DC Proof is the biggest teaching mistakeMostowski Collapse
| |             +- Re: DC Proof is the biggest teaching mistakeMostowski Collapse
| |             +- Re: DC Proof is the biggest teaching mistakeMostowski Collapse
| |             +- Re: DC Proof is the biggest teaching mistakeDan Christensen
| |             +- Re: DC Proof is the biggest teaching mistakeMostowski Collapse
| |             +- Re: DC Proof is the biggest teaching mistakeMostowski Collapse
| |             +- Re: DC Proof is the biggest teaching mistakeDan Christensen
| |             +- Re: DC Proof is the biggest teaching mistakeMostowski Collapse
| |             +- Re: DC Proof is the biggest teaching mistakeMostowski Collapse
| |             +- Re: DC Proof is the biggest teaching mistakeDan Christensen
| |             +- Re: DC Proof is the biggest teaching mistakeMostowski Collapse
| |             +- Re: DC Proof is the biggest teaching mistakeMostowski Collapse
| |             +- Re: DC Proof is the biggest teaching mistakeMostowski Collapse
| |             +- Re: DC Proof is the biggest teaching mistakeMostowski Collapse
| |             +- Re: DC Proof is the biggest teaching mistakeDan Christensen
| |             +- Re: DC Proof is the biggest teaching mistakeMostowski Collapse
| |             +- Re: DC Proof is the biggest teaching mistakeMostowski Collapse
| |             +- Re: DC Proof is the biggest teaching mistakeMostowski Collapse
| |             +- Re: DC Proof is the biggest teaching mistakeMostowski Collapse
| |             +- Re: DC Proof is the biggest teaching mistakeDan Christensen
| |             +- Re: DC Proof is the biggest teaching mistakeMostowski Collapse
| |             +- Re: DC Proof is the biggest teaching mistakeDan Christensen
| |             +- Re: DC Proof is the biggest teaching mistakeMostowski Collapse
| |             +- Re: DC Proof is the biggest teaching mistakeDan Christensen
| |             +- Re: DC Proof is the biggest teaching mistakeMostowski Collapse
| |             +- Re: DC Proof is the biggest teaching mistakeMostowski Collapse
| |             +- Re: DC Proof is the biggest teaching mistakeDan Christensen
| |             +- Re: DC Proof is the biggest teaching mistakeDan Christensen
| |             +- Re: DC Proof is the biggest teaching mistakeDan Christensen
| |             +- Re: DC Proof is the biggest teaching mistakeMostowski Collapse
| |             +- Re: DC Proof is the biggest teaching mistakeMostowski Collapse
| |             `- Re: DC Proof is the biggest teaching mistakeMostowski Collapse
| `* Re: DC Proof is the biggest teaching mistakeKip Foh
|  `* Re: DC Proof is the biggest teaching mistakeMostowski Collapse
|   `* Re: DC Proof is the biggest teaching mistakeKip Foh
|    `* Re: DC Proof is the biggest teaching mistakeMostowski Collapse
|     `- Re: DC Proof is the biggest teaching mistakeKip Foh
+* Re: DC Proof is the biggest teaching mistakeMostowski Collapse
|+* Re: DC Proof is the biggest teaching mistakeMostowski Collapse
||+- Re: DC Proof is the biggest teaching mistakeDan Christensen
||`* Re: DC Proof is the biggest teaching mistakeDan Christensen
|| `* Re: DC Proof is the biggest teaching mistakeMostowski Collapse
||  +- Re: DC Proof is the biggest teaching mistakeMostowski Collapse
||  `* Re: DC Proof is the biggest teaching mistakeDan Christensen
||   `* Re: DC Proof is the biggest teaching mistakeMostowski Collapse
||    `* Re: DC Proof is the biggest teaching mistakeMostowski Collapse
||     +- Re: DC Proof is the biggest teaching mistakeMostowski Collapse
||     `* Re: DC Proof is the biggest teaching mistakeDan Christensen
||      `* Re: DC Proof is the biggest teaching mistakeMostowski Collapse
||       `* Re: DC Proof is the biggest teaching mistakeDan Christensen
||        `* Re: DC Proof is the biggest teaching mistakeMostowski Collapse
||         +- Re: DC Proof is the biggest teaching mistakeMostowski Collapse
||         `* Re: DC Proof is the biggest teaching mistakeDan Christensen
||          `* Re: DC Proof is the biggest teaching mistakeMostowski Collapse
||           `- Re: DC Proof is the biggest teaching mistakeMostowski Collapse
|`* Re: DC Proof is the biggest teaching mistakeScot Dino
| `* Re: DC Proof is the biggest teaching mistakeMostowski Collapse
|  `* Re: DC Proof is the biggest teaching mistakemitchr...@gmail.com
|   `- Re: DC Proof is the biggest teaching mistakeMostowski Collapse
+* Re: DC Proof is the biggest teaching mistakeMostowski Collapse
|`* Re: DC Proof is the biggest teaching mistakeDan Christensen
+* Re: DC Proof is the biggest teaching mistakeMostowski Collapse
+* Re: DC Proof is the biggest teaching mistakeMostowski Collapse
+* Re: DC Proof is the biggest teaching mistakeMostowski Collapse
+* Re: DC Proof is the biggest teaching mistakeMostowski Collapse
+* Re: DC Proof is the biggest teaching mistakeMostowski Collapse
`* Re: DC Proof is the biggest teaching mistakeMostowski Collapse

Pages:12345678910111213141516
Re: DC Proof is the biggest teaching mistake

<c807383c-f994-4335-aef1-1149d9a6aaccn@googlegroups.com>

  copy mid

https://www.novabbs.com/tech/article-flat.php?id=88209&group=sci.math#88209

  copy link   Newsgroups: sci.math
X-Received: by 2002:a37:db12:: with SMTP id e18mr7355388qki.14.1642180828976;
Fri, 14 Jan 2022 09:20:28 -0800 (PST)
X-Received: by 2002:a25:cdc3:: with SMTP id d186mr14379481ybf.400.1642180828680;
Fri, 14 Jan 2022 09:20:28 -0800 (PST)
Path: i2pn2.org!i2pn.org!weretis.net!feeder6.news.weretis.net!1.us.feeder.erje.net!2.us.feeder.erje.net!feeder.erje.net!border1.nntp.dca1.giganews.com!nntp.giganews.com!news-out.google.com!nntp.google.com!postnews.google.com!google-groups.googlegroups.com!not-for-mail
Newsgroups: sci.math
Date: Fri, 14 Jan 2022 09:20:28 -0800 (PST)
In-Reply-To: <a42ad378-d7fb-48e1-8e94-4aab3f8f6ecan@googlegroups.com>
Injection-Info: google-groups.googlegroups.com; posting-host=128.208.191.236; posting-account=71XbuAoAAACx3_UV8yBrbgOAHUYjIUR6
NNTP-Posting-Host: 128.208.191.236
References: <bb7e1dba-3985-4555-8cf9-5c5421b69769n@googlegroups.com>
<sqvgk7$hqsh$1@solani.org> <e1dea604-6118-42a2-bc65-d1a1ffb75eecn@googlegroups.com>
<sqvhbb$hr88$2@solani.org> <411e3e45-29c7-4920-b775-0e0ef4e99f61n@googlegroups.com>
<89606698-34e1-46df-92e9-31abee426e10n@googlegroups.com> <4942e097-fe00-43ea-8d3e-24f3c6786b5dn@googlegroups.com>
<e2d289cc-e57e-4054-9544-6ae07f5bb585n@googlegroups.com> <b328f179-2765-47a4-9bec-53b3082fd847n@googlegroups.com>
<768dcb1f-80c4-457d-ac94-c89366b99ebfn@googlegroups.com> <77773d8c-4025-424c-8600-fc7ad6a922cen@googlegroups.com>
<354cd029-6017-4280-90ec-7c337be97bedn@googlegroups.com> <532b5c13-6122-4bef-a4ed-003df18cbdacn@googlegroups.com>
<4755f36f-9325-4326-b427-e25094c769dcn@googlegroups.com> <1bd61d43-8e19-4041-a050-89fb942262e6n@googlegroups.com>
<5557b6ad-6176-4dfe-81a6-758fd18813c4n@googlegroups.com> <52cc92cf-377b-4437-8d55-e933d4fba462n@googlegroups.com>
<1c2b577c-9b70-4479-ba85-c631f74079fdn@googlegroups.com> <2e6facbd-4202-4271-a8ea-2a2d1567aac8n@googlegroups.com>
<a75daafb-6adb-4d27-8650-09bcf995928fn@googlegroups.com> <8e3ad0b8-9951-4888-9593-4de90bba23d6n@googlegroups.com>
<173a8efd-28f8-4ec0-9a53-546d4bd4867an@googlegroups.com> <a64af915-4b00-4c8a-8249-47eb9b73147en@googlegroups.com>
<f097eeb2-9f31-4e0e-be3d-e4d370797c54n@googlegroups.com> <dc0b3a5a-1fa0-4729-9095-3545850cd5dcn@googlegroups.com>
<0b1bd903-87a5-4d36-a73a-ec0193b9a7e8n@googlegroups.com> <e24fc87c-feff-4493-81ca-060c9f676ec1n@googlegroups.com>
<05129406-d817-46bc-8935-ce5dbcd21a38n@googlegroups.com> <442fd60e-d3a2-4531-bb10-d2ac65552d82n@googlegroups.com>
<1304bb87-ebce-459e-a57f-e6bdf7dcebd8n@googlegroups.com> <a2df4b9b-322b-4f2e-a68f-04646fc94209n@googlegroups.com>
<ca406e7f-dc3e-4ee3-ad55-adedc315d698n@googlegroups.com> <86adee40-88d9-40df-acf6-5ed92b3dccfbn@googlegroups.com>
<20d4282a-6766-4089-ab1e-df4850c49733n@googlegroups.com> <f3fbe20c-797b-4b60-ad86-012f2d5d47f2n@googlegroups.com>
<615b2804-e5d9-4ab7-ae5e-e86c332b55e3n@googlegroups.com> <64d64634-f29e-42e6-a0cb-5230ae71afaan@googlegroups.com>
<b29ef605-a93e-49de-bec1-a6bdcc88f089n@googlegroups.com> <3b8e2323-473d-4031-8e23-9f92c5f3ea2fn@googlegroups.com>
<c1465a3d-f483-4088-ab45-ddca5e898a15n@googlegroups.com> <b41d5082-06db-4dfd-9d36-126683cba7fcn@googlegroups.com>
<95c67044-56f2-44f3-802c-575624de0281n@googlegroups.com> <bfad1477-322f-4e59-a9b2-9bd6ef2c9421n@googlegroups.com>
<0760211b-e4d4-4de3-8fd5-16c50c1b9251n@googlegroups.com> <a6120809-9a33-42fe-9590-4906f14345ddn@googlegroups.com>
<84f32020-acd7-481f-ac56-72755cce1a8an@googlegroups.com> <46991c1f-43b2-4806-b6fe-994faf80011dn@googlegroups.com>
<2f9be317-39b8-4516-992f-0d6b36bd324fn@googlegroups.com> <9149ed4f-04f6-4e88-b46f-f2da25f1c7aen@googlegroups.com>
<8eeea3c8-b5b1-4362-a4e9-bf005590a611n@googlegroups.com> <0b1410c9-f30b-4244-a326-c914e3f8825bn@googlegroups.com>
<6f5c7bca-ad0f-4daa-9424-91ae97e0fe1bn@googlegroups.com> <a56251eb-9116-4706-b692-8a6ebc2cfc13n@googlegroups.com>
<3a68a2e7-4251-4b07-ba93-2b873afd5d47n@googlegroups.com> <fa233e1d-7f3c-452e-8130-51ac7a9cde55n@googlegroups.com>
<a42ad378-d7fb-48e1-8e94-4aab3f8f6ecan@googlegroups.com>
User-Agent: G2/1.0
MIME-Version: 1.0
Message-ID: <c807383c-f994-4335-aef1-1149d9a6aaccn@googlegroups.com>
Subject: Re: DC Proof is the biggest teaching mistake
From: fredjeff...@gmail.com (FredJeffries)
Injection-Date: Fri, 14 Jan 2022 17:20:28 +0000
Content-Type: text/plain; charset="UTF-8"
Lines: 18
 by: FredJeffries - Fri, 14 Jan 2022 17:20 UTC

On Friday, January 14, 2022 at 9:04:25 AM UTC-8, Mostowski Collapse wrote:
> Well set theory, your prime enemy, solves the problem differently.
> When you use ordinals as for example von Neuman did when
> he was a youngster:
>
> This motivates the **standard** definition, suggested by John von
> Neumann at the age of 19, now called definition of von Neumann
> ordinals: "each ordinal is the well-ordered set of all smaller ordinals."
> https://en.wikipedia.org/wiki/Ordinal_number#Von_Neumann_definition_of_ordinals
>
> Then you have an easy canonical member inside set theory
> for every order type. In DC Proof for every arbitrary function f
> that is Dedekind infinite, there is some Peano structure,
>
> which one is canonical?

There is nothing 'canonical' about the von Neumann representation

https://en.wikipedia.org/wiki/Benacerraf%27s_identification_problem

Re: DC Proof is the biggest teaching mistake

<2751593d-e37e-4823-b05b-6f758dc72f96n@googlegroups.com>

  copy mid

https://www.novabbs.com/tech/article-flat.php?id=88210&group=sci.math#88210

  copy link   Newsgroups: sci.math
X-Received: by 2002:a05:6214:2a88:: with SMTP id jr8mr9021756qvb.118.1642180990449;
Fri, 14 Jan 2022 09:23:10 -0800 (PST)
X-Received: by 2002:a25:fd6:: with SMTP id 205mr13590230ybp.654.1642180990242;
Fri, 14 Jan 2022 09:23:10 -0800 (PST)
Path: i2pn2.org!i2pn.org!aioe.org!usenet.goja.nl.eu.org!3.eu.feeder.erje.net!feeder.erje.net!border1.nntp.dca1.giganews.com!nntp.giganews.com!news-out.google.com!nntp.google.com!postnews.google.com!google-groups.googlegroups.com!not-for-mail
Newsgroups: sci.math
Date: Fri, 14 Jan 2022 09:23:10 -0800 (PST)
In-Reply-To: <a42ad378-d7fb-48e1-8e94-4aab3f8f6ecan@googlegroups.com>
Injection-Info: google-groups.googlegroups.com; posting-host=163.182.226.42; posting-account=OWfgwwgAAADQpH2XgMDMe2wuQ7OFPXlE
NNTP-Posting-Host: 163.182.226.42
References: <bb7e1dba-3985-4555-8cf9-5c5421b69769n@googlegroups.com>
<sqvgk7$hqsh$1@solani.org> <e1dea604-6118-42a2-bc65-d1a1ffb75eecn@googlegroups.com>
<sqvhbb$hr88$2@solani.org> <411e3e45-29c7-4920-b775-0e0ef4e99f61n@googlegroups.com>
<89606698-34e1-46df-92e9-31abee426e10n@googlegroups.com> <4942e097-fe00-43ea-8d3e-24f3c6786b5dn@googlegroups.com>
<e2d289cc-e57e-4054-9544-6ae07f5bb585n@googlegroups.com> <b328f179-2765-47a4-9bec-53b3082fd847n@googlegroups.com>
<768dcb1f-80c4-457d-ac94-c89366b99ebfn@googlegroups.com> <77773d8c-4025-424c-8600-fc7ad6a922cen@googlegroups.com>
<354cd029-6017-4280-90ec-7c337be97bedn@googlegroups.com> <532b5c13-6122-4bef-a4ed-003df18cbdacn@googlegroups.com>
<4755f36f-9325-4326-b427-e25094c769dcn@googlegroups.com> <1bd61d43-8e19-4041-a050-89fb942262e6n@googlegroups.com>
<5557b6ad-6176-4dfe-81a6-758fd18813c4n@googlegroups.com> <52cc92cf-377b-4437-8d55-e933d4fba462n@googlegroups.com>
<1c2b577c-9b70-4479-ba85-c631f74079fdn@googlegroups.com> <2e6facbd-4202-4271-a8ea-2a2d1567aac8n@googlegroups.com>
<a75daafb-6adb-4d27-8650-09bcf995928fn@googlegroups.com> <8e3ad0b8-9951-4888-9593-4de90bba23d6n@googlegroups.com>
<173a8efd-28f8-4ec0-9a53-546d4bd4867an@googlegroups.com> <a64af915-4b00-4c8a-8249-47eb9b73147en@googlegroups.com>
<f097eeb2-9f31-4e0e-be3d-e4d370797c54n@googlegroups.com> <dc0b3a5a-1fa0-4729-9095-3545850cd5dcn@googlegroups.com>
<0b1bd903-87a5-4d36-a73a-ec0193b9a7e8n@googlegroups.com> <e24fc87c-feff-4493-81ca-060c9f676ec1n@googlegroups.com>
<05129406-d817-46bc-8935-ce5dbcd21a38n@googlegroups.com> <442fd60e-d3a2-4531-bb10-d2ac65552d82n@googlegroups.com>
<1304bb87-ebce-459e-a57f-e6bdf7dcebd8n@googlegroups.com> <a2df4b9b-322b-4f2e-a68f-04646fc94209n@googlegroups.com>
<ca406e7f-dc3e-4ee3-ad55-adedc315d698n@googlegroups.com> <86adee40-88d9-40df-acf6-5ed92b3dccfbn@googlegroups.com>
<20d4282a-6766-4089-ab1e-df4850c49733n@googlegroups.com> <f3fbe20c-797b-4b60-ad86-012f2d5d47f2n@googlegroups.com>
<615b2804-e5d9-4ab7-ae5e-e86c332b55e3n@googlegroups.com> <64d64634-f29e-42e6-a0cb-5230ae71afaan@googlegroups.com>
<b29ef605-a93e-49de-bec1-a6bdcc88f089n@googlegroups.com> <3b8e2323-473d-4031-8e23-9f92c5f3ea2fn@googlegroups.com>
<c1465a3d-f483-4088-ab45-ddca5e898a15n@googlegroups.com> <b41d5082-06db-4dfd-9d36-126683cba7fcn@googlegroups.com>
<95c67044-56f2-44f3-802c-575624de0281n@googlegroups.com> <bfad1477-322f-4e59-a9b2-9bd6ef2c9421n@googlegroups.com>
<0760211b-e4d4-4de3-8fd5-16c50c1b9251n@googlegroups.com> <a6120809-9a33-42fe-9590-4906f14345ddn@googlegroups.com>
<84f32020-acd7-481f-ac56-72755cce1a8an@googlegroups.com> <46991c1f-43b2-4806-b6fe-994faf80011dn@googlegroups.com>
<2f9be317-39b8-4516-992f-0d6b36bd324fn@googlegroups.com> <9149ed4f-04f6-4e88-b46f-f2da25f1c7aen@googlegroups.com>
<8eeea3c8-b5b1-4362-a4e9-bf005590a611n@googlegroups.com> <0b1410c9-f30b-4244-a326-c914e3f8825bn@googlegroups.com>
<6f5c7bca-ad0f-4daa-9424-91ae97e0fe1bn@googlegroups.com> <a56251eb-9116-4706-b692-8a6ebc2cfc13n@googlegroups.com>
<3a68a2e7-4251-4b07-ba93-2b873afd5d47n@googlegroups.com> <fa233e1d-7f3c-452e-8130-51ac7a9cde55n@googlegroups.com>
<a42ad378-d7fb-48e1-8e94-4aab3f8f6ecan@googlegroups.com>
User-Agent: G2/1.0
MIME-Version: 1.0
Message-ID: <2751593d-e37e-4823-b05b-6f758dc72f96n@googlegroups.com>
Subject: Re: DC Proof is the biggest teaching mistake
From: Dan_Chri...@sympatico.ca (Dan Christensen)
Injection-Date: Fri, 14 Jan 2022 17:23:10 +0000
Content-Type: text/plain; charset="UTF-8"
Content-Transfer-Encoding: quoted-printable
Lines: 21
 by: Dan Christensen - Fri, 14 Jan 2022 17:23 UTC

On Friday, January 14, 2022 at 12:04:25 PM UTC-5, Mostowski Collapse wrote:
> Well set theory, your prime enemy, solves the problem differently.
> When you use ordinals as for example von Neuman did when
> he was a youngster:
>
> This motivates the **standard** definition, suggested by John von
> Neumann at the age of 19, now called definition of von Neumann
> ordinals: "each ordinal is the well-ordered set of all smaller ordinals."
> https://en.wikipedia.org/wiki/Ordinal_number#Von_Neumann_definition_of_ordinals
>

Another pedagogical nightmare! In extremely few math textbooks will you find, for example, that 2 is a subset of 4 as at your link. Thanks, but not no thanks, Jan Burse. If students require axioms for the natural numbers, you can't beat simply starting with Peano's Axioms.

Dan

Download my DC Proof 2.0 freeware at http://www.dcproof.com
Visit my Math Blog at http://www.dcproof.wordpress.com

Re: DC Proof is the biggest teaching mistake

<855ecc7f-8e21-462d-bbb3-68788eda8529n@googlegroups.com>

  copy mid

https://www.novabbs.com/tech/article-flat.php?id=88211&group=sci.math#88211

  copy link   Newsgroups: sci.math
X-Received: by 2002:ad4:4ee6:: with SMTP id dv6mr8668634qvb.77.1642181093228;
Fri, 14 Jan 2022 09:24:53 -0800 (PST)
X-Received: by 2002:a25:46c1:: with SMTP id t184mr14240998yba.519.1642181092994;
Fri, 14 Jan 2022 09:24:52 -0800 (PST)
Path: i2pn2.org!i2pn.org!weretis.net!feeder6.news.weretis.net!news.misty.com!border2.nntp.dca1.giganews.com!border1.nntp.dca1.giganews.com!nntp.giganews.com!news-out.google.com!nntp.google.com!postnews.google.com!google-groups.googlegroups.com!not-for-mail
Newsgroups: sci.math
Date: Fri, 14 Jan 2022 09:24:52 -0800 (PST)
In-Reply-To: <c807383c-f994-4335-aef1-1149d9a6aaccn@googlegroups.com>
Injection-Info: google-groups.googlegroups.com; posting-host=77.57.53.70; posting-account=UjEXBwoAAAAOk5fiB8WdHvZddFg9nJ9r
NNTP-Posting-Host: 77.57.53.70
References: <bb7e1dba-3985-4555-8cf9-5c5421b69769n@googlegroups.com>
<sqvgk7$hqsh$1@solani.org> <e1dea604-6118-42a2-bc65-d1a1ffb75eecn@googlegroups.com>
<sqvhbb$hr88$2@solani.org> <411e3e45-29c7-4920-b775-0e0ef4e99f61n@googlegroups.com>
<89606698-34e1-46df-92e9-31abee426e10n@googlegroups.com> <4942e097-fe00-43ea-8d3e-24f3c6786b5dn@googlegroups.com>
<e2d289cc-e57e-4054-9544-6ae07f5bb585n@googlegroups.com> <b328f179-2765-47a4-9bec-53b3082fd847n@googlegroups.com>
<768dcb1f-80c4-457d-ac94-c89366b99ebfn@googlegroups.com> <77773d8c-4025-424c-8600-fc7ad6a922cen@googlegroups.com>
<354cd029-6017-4280-90ec-7c337be97bedn@googlegroups.com> <532b5c13-6122-4bef-a4ed-003df18cbdacn@googlegroups.com>
<4755f36f-9325-4326-b427-e25094c769dcn@googlegroups.com> <1bd61d43-8e19-4041-a050-89fb942262e6n@googlegroups.com>
<5557b6ad-6176-4dfe-81a6-758fd18813c4n@googlegroups.com> <52cc92cf-377b-4437-8d55-e933d4fba462n@googlegroups.com>
<1c2b577c-9b70-4479-ba85-c631f74079fdn@googlegroups.com> <2e6facbd-4202-4271-a8ea-2a2d1567aac8n@googlegroups.com>
<a75daafb-6adb-4d27-8650-09bcf995928fn@googlegroups.com> <8e3ad0b8-9951-4888-9593-4de90bba23d6n@googlegroups.com>
<173a8efd-28f8-4ec0-9a53-546d4bd4867an@googlegroups.com> <a64af915-4b00-4c8a-8249-47eb9b73147en@googlegroups.com>
<f097eeb2-9f31-4e0e-be3d-e4d370797c54n@googlegroups.com> <dc0b3a5a-1fa0-4729-9095-3545850cd5dcn@googlegroups.com>
<0b1bd903-87a5-4d36-a73a-ec0193b9a7e8n@googlegroups.com> <e24fc87c-feff-4493-81ca-060c9f676ec1n@googlegroups.com>
<05129406-d817-46bc-8935-ce5dbcd21a38n@googlegroups.com> <442fd60e-d3a2-4531-bb10-d2ac65552d82n@googlegroups.com>
<1304bb87-ebce-459e-a57f-e6bdf7dcebd8n@googlegroups.com> <a2df4b9b-322b-4f2e-a68f-04646fc94209n@googlegroups.com>
<ca406e7f-dc3e-4ee3-ad55-adedc315d698n@googlegroups.com> <86adee40-88d9-40df-acf6-5ed92b3dccfbn@googlegroups.com>
<20d4282a-6766-4089-ab1e-df4850c49733n@googlegroups.com> <f3fbe20c-797b-4b60-ad86-012f2d5d47f2n@googlegroups.com>
<615b2804-e5d9-4ab7-ae5e-e86c332b55e3n@googlegroups.com> <64d64634-f29e-42e6-a0cb-5230ae71afaan@googlegroups.com>
<b29ef605-a93e-49de-bec1-a6bdcc88f089n@googlegroups.com> <3b8e2323-473d-4031-8e23-9f92c5f3ea2fn@googlegroups.com>
<c1465a3d-f483-4088-ab45-ddca5e898a15n@googlegroups.com> <b41d5082-06db-4dfd-9d36-126683cba7fcn@googlegroups.com>
<95c67044-56f2-44f3-802c-575624de0281n@googlegroups.com> <bfad1477-322f-4e59-a9b2-9bd6ef2c9421n@googlegroups.com>
<0760211b-e4d4-4de3-8fd5-16c50c1b9251n@googlegroups.com> <a6120809-9a33-42fe-9590-4906f14345ddn@googlegroups.com>
<84f32020-acd7-481f-ac56-72755cce1a8an@googlegroups.com> <46991c1f-43b2-4806-b6fe-994faf80011dn@googlegroups.com>
<2f9be317-39b8-4516-992f-0d6b36bd324fn@googlegroups.com> <9149ed4f-04f6-4e88-b46f-f2da25f1c7aen@googlegroups.com>
<8eeea3c8-b5b1-4362-a4e9-bf005590a611n@googlegroups.com> <0b1410c9-f30b-4244-a326-c914e3f8825bn@googlegroups.com>
<6f5c7bca-ad0f-4daa-9424-91ae97e0fe1bn@googlegroups.com> <a56251eb-9116-4706-b692-8a6ebc2cfc13n@googlegroups.com>
<3a68a2e7-4251-4b07-ba93-2b873afd5d47n@googlegroups.com> <fa233e1d-7f3c-452e-8130-51ac7a9cde55n@googlegroups.com>
<a42ad378-d7fb-48e1-8e94-4aab3f8f6ecan@googlegroups.com> <c807383c-f994-4335-aef1-1149d9a6aaccn@googlegroups.com>
User-Agent: G2/1.0
MIME-Version: 1.0
Message-ID: <855ecc7f-8e21-462d-bbb3-68788eda8529n@googlegroups.com>
Subject: Re: DC Proof is the biggest teaching mistake
From: burse...@gmail.com (Mostowski Collapse)
Injection-Date: Fri, 14 Jan 2022 17:24:53 +0000
Content-Type: text/plain; charset="UTF-8"
Content-Transfer-Encoding: quoted-printable
Lines: 37
 by: Mostowski Collapse - Fri, 14 Jan 2022 17:24 UTC

Well you can prove the Mostowsky Collapse Lemma.
So inside a model, the transitive set B is unique:

* Mostowski Collapse Lemma: Suppose we have a set 𝐴
* and a relation 𝑅 on 𝐴 which is well-founded and extensional
* ("extensional" means: for all 𝑥,𝑦∈𝐴, if {𝑧∈𝐴∣𝑧𝑅𝑥}={𝑧∈𝐴∣𝑧𝑅𝑦},
* then 𝑥=𝑦 -- and a well-order is automatically extensional).
* Then there exists a unique transitive set 𝐵 such that (𝐴,𝑅)≃(𝐵,∈|𝐵×𝐵).

FredJeffries schrieb am Freitag, 14. Januar 2022 um 18:20:34 UTC+1:
> On Friday, January 14, 2022 at 9:04:25 AM UTC-8, Mostowski Collapse wrote:
> > Well set theory, your prime enemy, solves the problem differently.
> > When you use ordinals as for example von Neuman did when
> > he was a youngster:
> >
> > This motivates the **standard** definition, suggested by John von
> > Neumann at the age of 19, now called definition of von Neumann
> > ordinals: "each ordinal is the well-ordered set of all smaller ordinals.."
> > https://en.wikipedia.org/wiki/Ordinal_number#Von_Neumann_definition_of_ordinals
> >
> > Then you have an easy canonical member inside set theory
> > for every order type. In DC Proof for every arbitrary function f
> > that is Dedekind infinite, there is some Peano structure,
> >
> > which one is canonical?
> There is nothing 'canonical' about the von Neumann representation
>
> https://en.wikipedia.org/wiki/Benacerraf%27s_identification_problem

Re: DC Proof is the biggest teaching mistake

<ae89c82f-eb08-467f-aeaa-603ca99b5137n@googlegroups.com>

  copy mid

https://www.novabbs.com/tech/article-flat.php?id=88214&group=sci.math#88214

  copy link   Newsgroups: sci.math
X-Received: by 2002:a37:bd05:: with SMTP id n5mr7070761qkf.293.1642181414495;
Fri, 14 Jan 2022 09:30:14 -0800 (PST)
X-Received: by 2002:a25:98c6:: with SMTP id m6mr14276916ybo.494.1642181414297;
Fri, 14 Jan 2022 09:30:14 -0800 (PST)
Path: i2pn2.org!i2pn.org!eternal-september.org!reader02.eternal-september.org!border1.nntp.dca1.giganews.com!nntp.giganews.com!news-out.google.com!nntp.google.com!postnews.google.com!google-groups.googlegroups.com!not-for-mail
Newsgroups: sci.math
Date: Fri, 14 Jan 2022 09:30:14 -0800 (PST)
In-Reply-To: <855ecc7f-8e21-462d-bbb3-68788eda8529n@googlegroups.com>
Injection-Info: google-groups.googlegroups.com; posting-host=77.57.53.70; posting-account=UjEXBwoAAAAOk5fiB8WdHvZddFg9nJ9r
NNTP-Posting-Host: 77.57.53.70
References: <bb7e1dba-3985-4555-8cf9-5c5421b69769n@googlegroups.com>
<sqvgk7$hqsh$1@solani.org> <e1dea604-6118-42a2-bc65-d1a1ffb75eecn@googlegroups.com>
<sqvhbb$hr88$2@solani.org> <411e3e45-29c7-4920-b775-0e0ef4e99f61n@googlegroups.com>
<89606698-34e1-46df-92e9-31abee426e10n@googlegroups.com> <4942e097-fe00-43ea-8d3e-24f3c6786b5dn@googlegroups.com>
<e2d289cc-e57e-4054-9544-6ae07f5bb585n@googlegroups.com> <b328f179-2765-47a4-9bec-53b3082fd847n@googlegroups.com>
<768dcb1f-80c4-457d-ac94-c89366b99ebfn@googlegroups.com> <77773d8c-4025-424c-8600-fc7ad6a922cen@googlegroups.com>
<354cd029-6017-4280-90ec-7c337be97bedn@googlegroups.com> <532b5c13-6122-4bef-a4ed-003df18cbdacn@googlegroups.com>
<4755f36f-9325-4326-b427-e25094c769dcn@googlegroups.com> <1bd61d43-8e19-4041-a050-89fb942262e6n@googlegroups.com>
<5557b6ad-6176-4dfe-81a6-758fd18813c4n@googlegroups.com> <52cc92cf-377b-4437-8d55-e933d4fba462n@googlegroups.com>
<1c2b577c-9b70-4479-ba85-c631f74079fdn@googlegroups.com> <2e6facbd-4202-4271-a8ea-2a2d1567aac8n@googlegroups.com>
<a75daafb-6adb-4d27-8650-09bcf995928fn@googlegroups.com> <8e3ad0b8-9951-4888-9593-4de90bba23d6n@googlegroups.com>
<173a8efd-28f8-4ec0-9a53-546d4bd4867an@googlegroups.com> <a64af915-4b00-4c8a-8249-47eb9b73147en@googlegroups.com>
<f097eeb2-9f31-4e0e-be3d-e4d370797c54n@googlegroups.com> <dc0b3a5a-1fa0-4729-9095-3545850cd5dcn@googlegroups.com>
<0b1bd903-87a5-4d36-a73a-ec0193b9a7e8n@googlegroups.com> <e24fc87c-feff-4493-81ca-060c9f676ec1n@googlegroups.com>
<05129406-d817-46bc-8935-ce5dbcd21a38n@googlegroups.com> <442fd60e-d3a2-4531-bb10-d2ac65552d82n@googlegroups.com>
<1304bb87-ebce-459e-a57f-e6bdf7dcebd8n@googlegroups.com> <a2df4b9b-322b-4f2e-a68f-04646fc94209n@googlegroups.com>
<ca406e7f-dc3e-4ee3-ad55-adedc315d698n@googlegroups.com> <86adee40-88d9-40df-acf6-5ed92b3dccfbn@googlegroups.com>
<20d4282a-6766-4089-ab1e-df4850c49733n@googlegroups.com> <f3fbe20c-797b-4b60-ad86-012f2d5d47f2n@googlegroups.com>
<615b2804-e5d9-4ab7-ae5e-e86c332b55e3n@googlegroups.com> <64d64634-f29e-42e6-a0cb-5230ae71afaan@googlegroups.com>
<b29ef605-a93e-49de-bec1-a6bdcc88f089n@googlegroups.com> <3b8e2323-473d-4031-8e23-9f92c5f3ea2fn@googlegroups.com>
<c1465a3d-f483-4088-ab45-ddca5e898a15n@googlegroups.com> <b41d5082-06db-4dfd-9d36-126683cba7fcn@googlegroups.com>
<95c67044-56f2-44f3-802c-575624de0281n@googlegroups.com> <bfad1477-322f-4e59-a9b2-9bd6ef2c9421n@googlegroups.com>
<0760211b-e4d4-4de3-8fd5-16c50c1b9251n@googlegroups.com> <a6120809-9a33-42fe-9590-4906f14345ddn@googlegroups.com>
<84f32020-acd7-481f-ac56-72755cce1a8an@googlegroups.com> <46991c1f-43b2-4806-b6fe-994faf80011dn@googlegroups.com>
<2f9be317-39b8-4516-992f-0d6b36bd324fn@googlegroups.com> <9149ed4f-04f6-4e88-b46f-f2da25f1c7aen@googlegroups.com>
<8eeea3c8-b5b1-4362-a4e9-bf005590a611n@googlegroups.com> <0b1410c9-f30b-4244-a326-c914e3f8825bn@googlegroups.com>
<6f5c7bca-ad0f-4daa-9424-91ae97e0fe1bn@googlegroups.com> <a56251eb-9116-4706-b692-8a6ebc2cfc13n@googlegroups.com>
<3a68a2e7-4251-4b07-ba93-2b873afd5d47n@googlegroups.com> <fa233e1d-7f3c-452e-8130-51ac7a9cde55n@googlegroups.com>
<a42ad378-d7fb-48e1-8e94-4aab3f8f6ecan@googlegroups.com> <c807383c-f994-4335-aef1-1149d9a6aaccn@googlegroups.com>
<855ecc7f-8e21-462d-bbb3-68788eda8529n@googlegroups.com>
User-Agent: G2/1.0
MIME-Version: 1.0
Message-ID: <ae89c82f-eb08-467f-aeaa-603ca99b5137n@googlegroups.com>
Subject: Re: DC Proof is the biggest teaching mistake
From: burse...@gmail.com (Mostowski Collapse)
Injection-Date: Fri, 14 Jan 2022 17:30:14 +0000
Content-Type: text/plain; charset="UTF-8"
Content-Transfer-Encoding: quoted-printable
Lines: 59
 by: Mostowski Collapse - Fri, 14 Jan 2022 17:30 UTC

But you could also choose another representative.
The reason to choose von Neuman, and why its called
canonical, because its the **standard** in set theory:

So, the von Neumann ordinals get around the issues
with "too big" sets which cannot be constructed in ZFC,
as a "feature not a bug" of ZFC to avoid contradictions
such as Russell's paradox, or the Burali-Forti paradox
specifically related to the class of ordinal numbers (while
these issues are glossed over in naive set theory). Also,
having a representative of each well-order in which the
relation is gotten just from a basic term of the language
of ZFC has numerous technical advantages.
https://math.stackexchange.com/a/2976857/1002973

DC Proof hasn't spent more time than taking a shit
in the morning over many problems of numerosity
and ordering. Nevertheless Dan-O-Matik is the biggest

anti troll against WM and John Gabriel. Whereas
DC Proof is fake nonsense, there is still no proof
of existence of some natural numbers in DC Proof.

Mostowski Collapse schrieb am Freitag, 14. Januar 2022 um 18:24:59 UTC+1:
> Well you can prove the Mostowsky Collapse Lemma.
> So inside a model, the transitive set B is unique:
> * Mostowski Collapse Lemma: Suppose we have a set 𝐴
> * and a relation 𝑅 on 𝐴 which is well-founded and extensional
> * ("extensional" means: for all 𝑥,𝑦∈𝐴, if {𝑧∈𝐴∣𝑧𝑅𝑥}={𝑧∈𝐴∣𝑧𝑅𝑦},
> * then 𝑥=𝑦 -- and a well-order is automatically extensional).
> * Then there exists a unique transitive set 𝐵 such that (𝐴,𝑅)≃(𝐵,∈|𝐵×𝐵).
> FredJeffries schrieb am Freitag, 14. Januar 2022 um 18:20:34 UTC+1:
> > On Friday, January 14, 2022 at 9:04:25 AM UTC-8, Mostowski Collapse wrote:
> > > Well set theory, your prime enemy, solves the problem differently.
> > > When you use ordinals as for example von Neuman did when
> > > he was a youngster:
> > >
> > > This motivates the **standard** definition, suggested by John von
> > > Neumann at the age of 19, now called definition of von Neumann
> > > ordinals: "each ordinal is the well-ordered set of all smaller ordinals."
> > > https://en.wikipedia.org/wiki/Ordinal_number#Von_Neumann_definition_of_ordinals
> > >
> > > Then you have an easy canonical member inside set theory
> > > for every order type. In DC Proof for every arbitrary function f
> > > that is Dedekind infinite, there is some Peano structure,
> > >
> > > which one is canonical?
> > There is nothing 'canonical' about the von Neumann representation
> >
> > https://en.wikipedia.org/wiki/Benacerraf%27s_identification_problem

Re: DC Proof is the biggest teaching mistake

<7c453b2e-1514-47a7-8de1-9584c83a274an@googlegroups.com>

  copy mid

https://www.novabbs.com/tech/article-flat.php?id=88217&group=sci.math#88217

  copy link   Newsgroups: sci.math
X-Received: by 2002:a05:622a:308:: with SMTP id q8mr8426749qtw.463.1642181768833;
Fri, 14 Jan 2022 09:36:08 -0800 (PST)
X-Received: by 2002:a25:46c1:: with SMTP id t184mr14304474yba.519.1642181768579;
Fri, 14 Jan 2022 09:36:08 -0800 (PST)
Path: i2pn2.org!i2pn.org!weretis.net!feeder6.news.weretis.net!news.misty.com!border2.nntp.dca1.giganews.com!border1.nntp.dca1.giganews.com!nntp.giganews.com!news-out.google.com!nntp.google.com!postnews.google.com!google-groups.googlegroups.com!not-for-mail
Newsgroups: sci.math
Date: Fri, 14 Jan 2022 09:36:08 -0800 (PST)
In-Reply-To: <ae89c82f-eb08-467f-aeaa-603ca99b5137n@googlegroups.com>
Injection-Info: google-groups.googlegroups.com; posting-host=77.57.53.70; posting-account=UjEXBwoAAAAOk5fiB8WdHvZddFg9nJ9r
NNTP-Posting-Host: 77.57.53.70
References: <bb7e1dba-3985-4555-8cf9-5c5421b69769n@googlegroups.com>
<sqvgk7$hqsh$1@solani.org> <e1dea604-6118-42a2-bc65-d1a1ffb75eecn@googlegroups.com>
<sqvhbb$hr88$2@solani.org> <411e3e45-29c7-4920-b775-0e0ef4e99f61n@googlegroups.com>
<89606698-34e1-46df-92e9-31abee426e10n@googlegroups.com> <4942e097-fe00-43ea-8d3e-24f3c6786b5dn@googlegroups.com>
<e2d289cc-e57e-4054-9544-6ae07f5bb585n@googlegroups.com> <b328f179-2765-47a4-9bec-53b3082fd847n@googlegroups.com>
<768dcb1f-80c4-457d-ac94-c89366b99ebfn@googlegroups.com> <77773d8c-4025-424c-8600-fc7ad6a922cen@googlegroups.com>
<354cd029-6017-4280-90ec-7c337be97bedn@googlegroups.com> <532b5c13-6122-4bef-a4ed-003df18cbdacn@googlegroups.com>
<4755f36f-9325-4326-b427-e25094c769dcn@googlegroups.com> <1bd61d43-8e19-4041-a050-89fb942262e6n@googlegroups.com>
<5557b6ad-6176-4dfe-81a6-758fd18813c4n@googlegroups.com> <52cc92cf-377b-4437-8d55-e933d4fba462n@googlegroups.com>
<1c2b577c-9b70-4479-ba85-c631f74079fdn@googlegroups.com> <2e6facbd-4202-4271-a8ea-2a2d1567aac8n@googlegroups.com>
<a75daafb-6adb-4d27-8650-09bcf995928fn@googlegroups.com> <8e3ad0b8-9951-4888-9593-4de90bba23d6n@googlegroups.com>
<173a8efd-28f8-4ec0-9a53-546d4bd4867an@googlegroups.com> <a64af915-4b00-4c8a-8249-47eb9b73147en@googlegroups.com>
<f097eeb2-9f31-4e0e-be3d-e4d370797c54n@googlegroups.com> <dc0b3a5a-1fa0-4729-9095-3545850cd5dcn@googlegroups.com>
<0b1bd903-87a5-4d36-a73a-ec0193b9a7e8n@googlegroups.com> <e24fc87c-feff-4493-81ca-060c9f676ec1n@googlegroups.com>
<05129406-d817-46bc-8935-ce5dbcd21a38n@googlegroups.com> <442fd60e-d3a2-4531-bb10-d2ac65552d82n@googlegroups.com>
<1304bb87-ebce-459e-a57f-e6bdf7dcebd8n@googlegroups.com> <a2df4b9b-322b-4f2e-a68f-04646fc94209n@googlegroups.com>
<ca406e7f-dc3e-4ee3-ad55-adedc315d698n@googlegroups.com> <86adee40-88d9-40df-acf6-5ed92b3dccfbn@googlegroups.com>
<20d4282a-6766-4089-ab1e-df4850c49733n@googlegroups.com> <f3fbe20c-797b-4b60-ad86-012f2d5d47f2n@googlegroups.com>
<615b2804-e5d9-4ab7-ae5e-e86c332b55e3n@googlegroups.com> <64d64634-f29e-42e6-a0cb-5230ae71afaan@googlegroups.com>
<b29ef605-a93e-49de-bec1-a6bdcc88f089n@googlegroups.com> <3b8e2323-473d-4031-8e23-9f92c5f3ea2fn@googlegroups.com>
<c1465a3d-f483-4088-ab45-ddca5e898a15n@googlegroups.com> <b41d5082-06db-4dfd-9d36-126683cba7fcn@googlegroups.com>
<95c67044-56f2-44f3-802c-575624de0281n@googlegroups.com> <bfad1477-322f-4e59-a9b2-9bd6ef2c9421n@googlegroups.com>
<0760211b-e4d4-4de3-8fd5-16c50c1b9251n@googlegroups.com> <a6120809-9a33-42fe-9590-4906f14345ddn@googlegroups.com>
<84f32020-acd7-481f-ac56-72755cce1a8an@googlegroups.com> <46991c1f-43b2-4806-b6fe-994faf80011dn@googlegroups.com>
<2f9be317-39b8-4516-992f-0d6b36bd324fn@googlegroups.com> <9149ed4f-04f6-4e88-b46f-f2da25f1c7aen@googlegroups.com>
<8eeea3c8-b5b1-4362-a4e9-bf005590a611n@googlegroups.com> <0b1410c9-f30b-4244-a326-c914e3f8825bn@googlegroups.com>
<6f5c7bca-ad0f-4daa-9424-91ae97e0fe1bn@googlegroups.com> <a56251eb-9116-4706-b692-8a6ebc2cfc13n@googlegroups.com>
<3a68a2e7-4251-4b07-ba93-2b873afd5d47n@googlegroups.com> <fa233e1d-7f3c-452e-8130-51ac7a9cde55n@googlegroups.com>
<a42ad378-d7fb-48e1-8e94-4aab3f8f6ecan@googlegroups.com> <c807383c-f994-4335-aef1-1149d9a6aaccn@googlegroups.com>
<855ecc7f-8e21-462d-bbb3-68788eda8529n@googlegroups.com> <ae89c82f-eb08-467f-aeaa-603ca99b5137n@googlegroups.com>
User-Agent: G2/1.0
MIME-Version: 1.0
Message-ID: <7c453b2e-1514-47a7-8de1-9584c83a274an@googlegroups.com>
Subject: Re: DC Proof is the biggest teaching mistake
From: burse...@gmail.com (Mostowski Collapse)
Injection-Date: Fri, 14 Jan 2022 17:36:08 +0000
Content-Type: text/plain; charset="UTF-8"
Content-Transfer-Encoding: quoted-printable
Lines: 79
 by: Mostowski Collapse - Fri, 14 Jan 2022 17:36 UTC

Dan-O-Matik doesn't choose some representation.
Well we could say he doesn't have to do so. He could
also work with an equivalence class, or like

for example the class of all Peano structures.
But Dan-O-Matik cannot prove that the class of
Peano structures is non-empty. On the otherhand

in set theory the proof of a representative element c of
course implies also that the corresponding class
C is non-empty, since alas the representative element

is member of the class, c e C. So in the worst case
the class is only {c}, possibly bigger, but it is non-empty.

LoL

Mostowski Collapse schrieb am Freitag, 14. Januar 2022 um 18:30:21 UTC+1:
> But you could also choose another representative.
> The reason to choose von Neuman, and why its called
> canonical, because its the **standard** in set theory:
>
> So, the von Neumann ordinals get around the issues
> with "too big" sets which cannot be constructed in ZFC,
> as a "feature not a bug" of ZFC to avoid contradictions
> such as Russell's paradox, or the Burali-Forti paradox
> specifically related to the class of ordinal numbers (while
> these issues are glossed over in naive set theory). Also,
> having a representative of each well-order in which the
> relation is gotten just from a basic term of the language
> of ZFC has numerous technical advantages.
> https://math.stackexchange.com/a/2976857/1002973
>
> DC Proof hasn't spent more time than taking a shit
> in the morning over many problems of numerosity
> and ordering. Nevertheless Dan-O-Matik is the biggest
>
> anti troll against WM and John Gabriel. Whereas
> DC Proof is fake nonsense, there is still no proof
> of existence of some natural numbers in DC Proof.
> Mostowski Collapse schrieb am Freitag, 14. Januar 2022 um 18:24:59 UTC+1:
> > Well you can prove the Mostowsky Collapse Lemma.
> > So inside a model, the transitive set B is unique:
> > * Mostowski Collapse Lemma: Suppose we have a set 𝐴
> > * and a relation 𝑅 on 𝐴 which is well-founded and extensional
> > * ("extensional" means: for all 𝑥,𝑦∈𝐴, if {𝑧∈𝐴∣𝑧𝑅𝑥}={𝑧∈𝐴∣𝑧𝑅𝑦},
> > * then 𝑥=𝑦 -- and a well-order is automatically extensional).
> > * Then there exists a unique transitive set 𝐵 such that (𝐴,𝑅)≃(𝐵,∈|𝐵×𝐵).
> > FredJeffries schrieb am Freitag, 14. Januar 2022 um 18:20:34 UTC+1:
> > > On Friday, January 14, 2022 at 9:04:25 AM UTC-8, Mostowski Collapse wrote:
> > > > Well set theory, your prime enemy, solves the problem differently.
> > > > When you use ordinals as for example von Neuman did when
> > > > he was a youngster:
> > > >
> > > > This motivates the **standard** definition, suggested by John von
> > > > Neumann at the age of 19, now called definition of von Neumann
> > > > ordinals: "each ordinal is the well-ordered set of all smaller ordinals."
> > > > https://en.wikipedia.org/wiki/Ordinal_number#Von_Neumann_definition_of_ordinals
> > > >
> > > > Then you have an easy canonical member inside set theory
> > > > for every order type. In DC Proof for every arbitrary function f
> > > > that is Dedekind infinite, there is some Peano structure,
> > > >
> > > > which one is canonical?
> > > There is nothing 'canonical' about the von Neumann representation
> > >
> > > https://en.wikipedia.org/wiki/Benacerraf%27s_identification_problem

Re: DC Proof is the biggest teaching mistake

<cd25961b-2b5a-402b-8149-b24713ba5831n@googlegroups.com>

  copy mid

https://www.novabbs.com/tech/article-flat.php?id=88218&group=sci.math#88218

  copy link   Newsgroups: sci.math
X-Received: by 2002:ac8:5845:: with SMTP id h5mr8757578qth.365.1642182239728;
Fri, 14 Jan 2022 09:43:59 -0800 (PST)
X-Received: by 2002:a5b:808:: with SMTP id x8mr15769126ybp.663.1642182239555;
Fri, 14 Jan 2022 09:43:59 -0800 (PST)
Path: i2pn2.org!i2pn.org!weretis.net!feeder6.news.weretis.net!news.misty.com!border2.nntp.dca1.giganews.com!border1.nntp.dca1.giganews.com!nntp.giganews.com!news-out.google.com!nntp.google.com!postnews.google.com!google-groups.googlegroups.com!not-for-mail
Newsgroups: sci.math
Date: Fri, 14 Jan 2022 09:43:59 -0800 (PST)
In-Reply-To: <7c453b2e-1514-47a7-8de1-9584c83a274an@googlegroups.com>
Injection-Info: google-groups.googlegroups.com; posting-host=77.57.53.70; posting-account=UjEXBwoAAAAOk5fiB8WdHvZddFg9nJ9r
NNTP-Posting-Host: 77.57.53.70
References: <bb7e1dba-3985-4555-8cf9-5c5421b69769n@googlegroups.com>
<sqvgk7$hqsh$1@solani.org> <e1dea604-6118-42a2-bc65-d1a1ffb75eecn@googlegroups.com>
<sqvhbb$hr88$2@solani.org> <411e3e45-29c7-4920-b775-0e0ef4e99f61n@googlegroups.com>
<89606698-34e1-46df-92e9-31abee426e10n@googlegroups.com> <4942e097-fe00-43ea-8d3e-24f3c6786b5dn@googlegroups.com>
<e2d289cc-e57e-4054-9544-6ae07f5bb585n@googlegroups.com> <b328f179-2765-47a4-9bec-53b3082fd847n@googlegroups.com>
<768dcb1f-80c4-457d-ac94-c89366b99ebfn@googlegroups.com> <77773d8c-4025-424c-8600-fc7ad6a922cen@googlegroups.com>
<354cd029-6017-4280-90ec-7c337be97bedn@googlegroups.com> <532b5c13-6122-4bef-a4ed-003df18cbdacn@googlegroups.com>
<4755f36f-9325-4326-b427-e25094c769dcn@googlegroups.com> <1bd61d43-8e19-4041-a050-89fb942262e6n@googlegroups.com>
<5557b6ad-6176-4dfe-81a6-758fd18813c4n@googlegroups.com> <52cc92cf-377b-4437-8d55-e933d4fba462n@googlegroups.com>
<1c2b577c-9b70-4479-ba85-c631f74079fdn@googlegroups.com> <2e6facbd-4202-4271-a8ea-2a2d1567aac8n@googlegroups.com>
<a75daafb-6adb-4d27-8650-09bcf995928fn@googlegroups.com> <8e3ad0b8-9951-4888-9593-4de90bba23d6n@googlegroups.com>
<173a8efd-28f8-4ec0-9a53-546d4bd4867an@googlegroups.com> <a64af915-4b00-4c8a-8249-47eb9b73147en@googlegroups.com>
<f097eeb2-9f31-4e0e-be3d-e4d370797c54n@googlegroups.com> <dc0b3a5a-1fa0-4729-9095-3545850cd5dcn@googlegroups.com>
<0b1bd903-87a5-4d36-a73a-ec0193b9a7e8n@googlegroups.com> <e24fc87c-feff-4493-81ca-060c9f676ec1n@googlegroups.com>
<05129406-d817-46bc-8935-ce5dbcd21a38n@googlegroups.com> <442fd60e-d3a2-4531-bb10-d2ac65552d82n@googlegroups.com>
<1304bb87-ebce-459e-a57f-e6bdf7dcebd8n@googlegroups.com> <a2df4b9b-322b-4f2e-a68f-04646fc94209n@googlegroups.com>
<ca406e7f-dc3e-4ee3-ad55-adedc315d698n@googlegroups.com> <86adee40-88d9-40df-acf6-5ed92b3dccfbn@googlegroups.com>
<20d4282a-6766-4089-ab1e-df4850c49733n@googlegroups.com> <f3fbe20c-797b-4b60-ad86-012f2d5d47f2n@googlegroups.com>
<615b2804-e5d9-4ab7-ae5e-e86c332b55e3n@googlegroups.com> <64d64634-f29e-42e6-a0cb-5230ae71afaan@googlegroups.com>
<b29ef605-a93e-49de-bec1-a6bdcc88f089n@googlegroups.com> <3b8e2323-473d-4031-8e23-9f92c5f3ea2fn@googlegroups.com>
<c1465a3d-f483-4088-ab45-ddca5e898a15n@googlegroups.com> <b41d5082-06db-4dfd-9d36-126683cba7fcn@googlegroups.com>
<95c67044-56f2-44f3-802c-575624de0281n@googlegroups.com> <bfad1477-322f-4e59-a9b2-9bd6ef2c9421n@googlegroups.com>
<0760211b-e4d4-4de3-8fd5-16c50c1b9251n@googlegroups.com> <a6120809-9a33-42fe-9590-4906f14345ddn@googlegroups.com>
<84f32020-acd7-481f-ac56-72755cce1a8an@googlegroups.com> <46991c1f-43b2-4806-b6fe-994faf80011dn@googlegroups.com>
<2f9be317-39b8-4516-992f-0d6b36bd324fn@googlegroups.com> <9149ed4f-04f6-4e88-b46f-f2da25f1c7aen@googlegroups.com>
<8eeea3c8-b5b1-4362-a4e9-bf005590a611n@googlegroups.com> <0b1410c9-f30b-4244-a326-c914e3f8825bn@googlegroups.com>
<6f5c7bca-ad0f-4daa-9424-91ae97e0fe1bn@googlegroups.com> <a56251eb-9116-4706-b692-8a6ebc2cfc13n@googlegroups.com>
<3a68a2e7-4251-4b07-ba93-2b873afd5d47n@googlegroups.com> <fa233e1d-7f3c-452e-8130-51ac7a9cde55n@googlegroups.com>
<a42ad378-d7fb-48e1-8e94-4aab3f8f6ecan@googlegroups.com> <c807383c-f994-4335-aef1-1149d9a6aaccn@googlegroups.com>
<855ecc7f-8e21-462d-bbb3-68788eda8529n@googlegroups.com> <ae89c82f-eb08-467f-aeaa-603ca99b5137n@googlegroups.com>
<7c453b2e-1514-47a7-8de1-9584c83a274an@googlegroups.com>
User-Agent: G2/1.0
MIME-Version: 1.0
Message-ID: <cd25961b-2b5a-402b-8149-b24713ba5831n@googlegroups.com>
Subject: Re: DC Proof is the biggest teaching mistake
From: burse...@gmail.com (Mostowski Collapse)
Injection-Date: Fri, 14 Jan 2022 17:43:59 +0000
Content-Type: text/plain; charset="UTF-8"
Content-Transfer-Encoding: quoted-printable
Lines: 110
 by: Mostowski Collapse - Fri, 14 Jan 2022 17:43 UTC

Dan-O-Matik halucinated:
> that 2 is a subset of 4 as at your link

You don't have to choose 2 and 4 this way.
You can also choose differently. The main point
would be to show that this collection exists:

{0,1,2,3,...}

Can you show existence of natural numbers in DC Proof.
In set theory omega exists so the class of infinite countable
well ordered sets is non empty.

Is the class of your Peano structures non-empty? I
doubt that you can executed the proof, because of
your recent changes. You cannot prove anymore:

> This is the proof that worked in dcproof2.exe, but
> does not work anymore in dcproof5.exe:
> [...]
> 8 ~Field(constr(a))
> U Spec, 6, 7
https://groups.google.com/g/sci.math/c/SOIECV0E-Dc/m/LGiS4I8UAwAJ

My speculation, technically you possibly cannot executed
the required proof anymore in DC Proof.
Mostowski Collapse schrieb am Freitag, 14. Januar 2022 um 18:36:15 UTC+1:
> Dan-O-Matik doesn't choose some representation.
> Well we could say he doesn't have to do so. He could
> also work with an equivalence class, or like
>
> for example the class of all Peano structures.
> But Dan-O-Matik cannot prove that the class of
> Peano structures is non-empty. On the otherhand
>
> in set theory the proof of a representative element c of
> course implies also that the corresponding class
> C is non-empty, since alas the representative element
>
> is member of the class, c e C. So in the worst case
> the class is only {c}, possibly bigger, but it is non-empty.
>
> LoL
> Mostowski Collapse schrieb am Freitag, 14. Januar 2022 um 18:30:21 UTC+1:
> > But you could also choose another representative.
> > The reason to choose von Neuman, and why its called
> > canonical, because its the **standard** in set theory:
> >
> > So, the von Neumann ordinals get around the issues
> > with "too big" sets which cannot be constructed in ZFC,
> > as a "feature not a bug" of ZFC to avoid contradictions
> > such as Russell's paradox, or the Burali-Forti paradox
> > specifically related to the class of ordinal numbers (while
> > these issues are glossed over in naive set theory). Also,
> > having a representative of each well-order in which the
> > relation is gotten just from a basic term of the language
> > of ZFC has numerous technical advantages.
> > https://math.stackexchange.com/a/2976857/1002973
> >
> > DC Proof hasn't spent more time than taking a shit
> > in the morning over many problems of numerosity
> > and ordering. Nevertheless Dan-O-Matik is the biggest
> >
> > anti troll against WM and John Gabriel. Whereas
> > DC Proof is fake nonsense, there is still no proof
> > of existence of some natural numbers in DC Proof.
> > Mostowski Collapse schrieb am Freitag, 14. Januar 2022 um 18:24:59 UTC+1:
> > > Well you can prove the Mostowsky Collapse Lemma.
> > > So inside a model, the transitive set B is unique:
> > > * Mostowski Collapse Lemma: Suppose we have a set 𝐴
> > > * and a relation 𝑅 on 𝐴 which is well-founded and extensional
> > > * ("extensional" means: for all 𝑥,𝑦∈𝐴, if {𝑧∈𝐴∣𝑧𝑅𝑥}={𝑧∈𝐴∣𝑧𝑅𝑦},
> > > * then 𝑥=𝑦 -- and a well-order is automatically extensional).
> > > * Then there exists a unique transitive set 𝐵 such that (𝐴,𝑅)≃(𝐵,∈|𝐵×𝐵).
> > > FredJeffries schrieb am Freitag, 14. Januar 2022 um 18:20:34 UTC+1:
> > > > On Friday, January 14, 2022 at 9:04:25 AM UTC-8, Mostowski Collapse wrote:
> > > > > Well set theory, your prime enemy, solves the problem differently..
> > > > > When you use ordinals as for example von Neuman did when
> > > > > he was a youngster:
> > > > >
> > > > > This motivates the **standard** definition, suggested by John von
> > > > > Neumann at the age of 19, now called definition of von Neumann
> > > > > ordinals: "each ordinal is the well-ordered set of all smaller ordinals."
> > > > > https://en.wikipedia.org/wiki/Ordinal_number#Von_Neumann_definition_of_ordinals
> > > > >
> > > > > Then you have an easy canonical member inside set theory
> > > > > for every order type. In DC Proof for every arbitrary function f
> > > > > that is Dedekind infinite, there is some Peano structure,
> > > > >
> > > > > which one is canonical?
> > > > There is nothing 'canonical' about the von Neumann representation
> > > >
> > > > https://en.wikipedia.org/wiki/Benacerraf%27s_identification_problem

Re: DC Proof is the biggest teaching mistake

<1746e145-6dbe-42d8-88bc-12db494b2374n@googlegroups.com>

  copy mid

https://www.novabbs.com/tech/article-flat.php?id=88219&group=sci.math#88219

  copy link   Newsgroups: sci.math
X-Received: by 2002:a05:622a:1787:: with SMTP id s7mr8449434qtk.550.1642182343010;
Fri, 14 Jan 2022 09:45:43 -0800 (PST)
X-Received: by 2002:a25:46c1:: with SMTP id t184mr14358688yba.519.1642182342808;
Fri, 14 Jan 2022 09:45:42 -0800 (PST)
Path: i2pn2.org!i2pn.org!usenet.goja.nl.eu.org!2.eu.feeder.erje.net!feeder.erje.net!border1.nntp.dca1.giganews.com!nntp.giganews.com!news-out.google.com!nntp.google.com!postnews.google.com!google-groups.googlegroups.com!not-for-mail
Newsgroups: sci.math
Date: Fri, 14 Jan 2022 09:45:42 -0800 (PST)
In-Reply-To: <cd25961b-2b5a-402b-8149-b24713ba5831n@googlegroups.com>
Injection-Info: google-groups.googlegroups.com; posting-host=77.57.53.70; posting-account=UjEXBwoAAAAOk5fiB8WdHvZddFg9nJ9r
NNTP-Posting-Host: 77.57.53.70
References: <bb7e1dba-3985-4555-8cf9-5c5421b69769n@googlegroups.com>
<sqvgk7$hqsh$1@solani.org> <e1dea604-6118-42a2-bc65-d1a1ffb75eecn@googlegroups.com>
<sqvhbb$hr88$2@solani.org> <411e3e45-29c7-4920-b775-0e0ef4e99f61n@googlegroups.com>
<89606698-34e1-46df-92e9-31abee426e10n@googlegroups.com> <4942e097-fe00-43ea-8d3e-24f3c6786b5dn@googlegroups.com>
<e2d289cc-e57e-4054-9544-6ae07f5bb585n@googlegroups.com> <b328f179-2765-47a4-9bec-53b3082fd847n@googlegroups.com>
<768dcb1f-80c4-457d-ac94-c89366b99ebfn@googlegroups.com> <77773d8c-4025-424c-8600-fc7ad6a922cen@googlegroups.com>
<354cd029-6017-4280-90ec-7c337be97bedn@googlegroups.com> <532b5c13-6122-4bef-a4ed-003df18cbdacn@googlegroups.com>
<4755f36f-9325-4326-b427-e25094c769dcn@googlegroups.com> <1bd61d43-8e19-4041-a050-89fb942262e6n@googlegroups.com>
<5557b6ad-6176-4dfe-81a6-758fd18813c4n@googlegroups.com> <52cc92cf-377b-4437-8d55-e933d4fba462n@googlegroups.com>
<1c2b577c-9b70-4479-ba85-c631f74079fdn@googlegroups.com> <2e6facbd-4202-4271-a8ea-2a2d1567aac8n@googlegroups.com>
<a75daafb-6adb-4d27-8650-09bcf995928fn@googlegroups.com> <8e3ad0b8-9951-4888-9593-4de90bba23d6n@googlegroups.com>
<173a8efd-28f8-4ec0-9a53-546d4bd4867an@googlegroups.com> <a64af915-4b00-4c8a-8249-47eb9b73147en@googlegroups.com>
<f097eeb2-9f31-4e0e-be3d-e4d370797c54n@googlegroups.com> <dc0b3a5a-1fa0-4729-9095-3545850cd5dcn@googlegroups.com>
<0b1bd903-87a5-4d36-a73a-ec0193b9a7e8n@googlegroups.com> <e24fc87c-feff-4493-81ca-060c9f676ec1n@googlegroups.com>
<05129406-d817-46bc-8935-ce5dbcd21a38n@googlegroups.com> <442fd60e-d3a2-4531-bb10-d2ac65552d82n@googlegroups.com>
<1304bb87-ebce-459e-a57f-e6bdf7dcebd8n@googlegroups.com> <a2df4b9b-322b-4f2e-a68f-04646fc94209n@googlegroups.com>
<ca406e7f-dc3e-4ee3-ad55-adedc315d698n@googlegroups.com> <86adee40-88d9-40df-acf6-5ed92b3dccfbn@googlegroups.com>
<20d4282a-6766-4089-ab1e-df4850c49733n@googlegroups.com> <f3fbe20c-797b-4b60-ad86-012f2d5d47f2n@googlegroups.com>
<615b2804-e5d9-4ab7-ae5e-e86c332b55e3n@googlegroups.com> <64d64634-f29e-42e6-a0cb-5230ae71afaan@googlegroups.com>
<b29ef605-a93e-49de-bec1-a6bdcc88f089n@googlegroups.com> <3b8e2323-473d-4031-8e23-9f92c5f3ea2fn@googlegroups.com>
<c1465a3d-f483-4088-ab45-ddca5e898a15n@googlegroups.com> <b41d5082-06db-4dfd-9d36-126683cba7fcn@googlegroups.com>
<95c67044-56f2-44f3-802c-575624de0281n@googlegroups.com> <bfad1477-322f-4e59-a9b2-9bd6ef2c9421n@googlegroups.com>
<0760211b-e4d4-4de3-8fd5-16c50c1b9251n@googlegroups.com> <a6120809-9a33-42fe-9590-4906f14345ddn@googlegroups.com>
<84f32020-acd7-481f-ac56-72755cce1a8an@googlegroups.com> <46991c1f-43b2-4806-b6fe-994faf80011dn@googlegroups.com>
<2f9be317-39b8-4516-992f-0d6b36bd324fn@googlegroups.com> <9149ed4f-04f6-4e88-b46f-f2da25f1c7aen@googlegroups.com>
<8eeea3c8-b5b1-4362-a4e9-bf005590a611n@googlegroups.com> <0b1410c9-f30b-4244-a326-c914e3f8825bn@googlegroups.com>
<6f5c7bca-ad0f-4daa-9424-91ae97e0fe1bn@googlegroups.com> <a56251eb-9116-4706-b692-8a6ebc2cfc13n@googlegroups.com>
<3a68a2e7-4251-4b07-ba93-2b873afd5d47n@googlegroups.com> <fa233e1d-7f3c-452e-8130-51ac7a9cde55n@googlegroups.com>
<a42ad378-d7fb-48e1-8e94-4aab3f8f6ecan@googlegroups.com> <c807383c-f994-4335-aef1-1149d9a6aaccn@googlegroups.com>
<855ecc7f-8e21-462d-bbb3-68788eda8529n@googlegroups.com> <ae89c82f-eb08-467f-aeaa-603ca99b5137n@googlegroups.com>
<7c453b2e-1514-47a7-8de1-9584c83a274an@googlegroups.com> <cd25961b-2b5a-402b-8149-b24713ba5831n@googlegroups.com>
User-Agent: G2/1.0
MIME-Version: 1.0
Message-ID: <1746e145-6dbe-42d8-88bc-12db494b2374n@googlegroups.com>
Subject: Re: DC Proof is the biggest teaching mistake
From: burse...@gmail.com (Mostowski Collapse)
Injection-Date: Fri, 14 Jan 2022 17:45:43 +0000
Content-Type: text/plain; charset="UTF-8"
Content-Transfer-Encoding: quoted-printable
Lines: 117
 by: Mostowski Collapse - Fri, 14 Jan 2022 17:45 UTC

Corr.:

Can you show existence of natural numbers in DC Proof.
In set theory omega exists so the class of infinite countable
well ordered sets without limit points is non empty.

Mostowski Collapse schrieb am Freitag, 14. Januar 2022 um 18:44:05 UTC+1:
> Dan-O-Matik halucinated:
> > that 2 is a subset of 4 as at your link
> You don't have to choose 2 and 4 this way.
> You can also choose differently. The main point
> would be to show that this collection exists:
>
> {0,1,2,3,...}
>
> Can you show existence of natural numbers in DC Proof.
> In set theory omega exists so the class of infinite countable
> well ordered sets is non empty.
>
> Is the class of your Peano structures non-empty? I
> doubt that you can executed the proof, because of
> your recent changes. You cannot prove anymore:
> > This is the proof that worked in dcproof2.exe, but
> > does not work anymore in dcproof5.exe:
> > [...]
> > 8 ~Field(constr(a))
> > U Spec, 6, 7
> https://groups.google.com/g/sci.math/c/SOIECV0E-Dc/m/LGiS4I8UAwAJ
>
> My speculation, technically you possibly cannot executed
> the required proof anymore in DC Proof.
> Mostowski Collapse schrieb am Freitag, 14. Januar 2022 um 18:36:15 UTC+1:
> > Dan-O-Matik doesn't choose some representation.
> > Well we could say he doesn't have to do so. He could
> > also work with an equivalence class, or like
> >
> > for example the class of all Peano structures.
> > But Dan-O-Matik cannot prove that the class of
> > Peano structures is non-empty. On the otherhand
> >
> > in set theory the proof of a representative element c of
> > course implies also that the corresponding class
> > C is non-empty, since alas the representative element
> >
> > is member of the class, c e C. So in the worst case
> > the class is only {c}, possibly bigger, but it is non-empty.
> >
> > LoL
> > Mostowski Collapse schrieb am Freitag, 14. Januar 2022 um 18:30:21 UTC+1:
> > > But you could also choose another representative.
> > > The reason to choose von Neuman, and why its called
> > > canonical, because its the **standard** in set theory:
> > >
> > > So, the von Neumann ordinals get around the issues
> > > with "too big" sets which cannot be constructed in ZFC,
> > > as a "feature not a bug" of ZFC to avoid contradictions
> > > such as Russell's paradox, or the Burali-Forti paradox
> > > specifically related to the class of ordinal numbers (while
> > > these issues are glossed over in naive set theory). Also,
> > > having a representative of each well-order in which the
> > > relation is gotten just from a basic term of the language
> > > of ZFC has numerous technical advantages.
> > > https://math.stackexchange.com/a/2976857/1002973
> > >
> > > DC Proof hasn't spent more time than taking a shit
> > > in the morning over many problems of numerosity
> > > and ordering. Nevertheless Dan-O-Matik is the biggest
> > >
> > > anti troll against WM and John Gabriel. Whereas
> > > DC Proof is fake nonsense, there is still no proof
> > > of existence of some natural numbers in DC Proof.
> > > Mostowski Collapse schrieb am Freitag, 14. Januar 2022 um 18:24:59 UTC+1:
> > > > Well you can prove the Mostowsky Collapse Lemma.
> > > > So inside a model, the transitive set B is unique:
> > > > * Mostowski Collapse Lemma: Suppose we have a set 𝐴
> > > > * and a relation 𝑅 on 𝐴 which is well-founded and extensional
> > > > * ("extensional" means: for all 𝑥,𝑦∈𝐴, if {𝑧∈𝐴∣𝑧𝑅𝑥}={𝑧∈𝐴∣𝑧𝑅𝑦},
> > > > * then 𝑥=𝑦 -- and a well-order is automatically extensional).
> > > > * Then there exists a unique transitive set 𝐵 such that (𝐴,𝑅)≃(𝐵,∈|𝐵×𝐵).
> > > > FredJeffries schrieb am Freitag, 14. Januar 2022 um 18:20:34 UTC+1:
> > > > > On Friday, January 14, 2022 at 9:04:25 AM UTC-8, Mostowski Collapse wrote:
> > > > > > Well set theory, your prime enemy, solves the problem differently.
> > > > > > When you use ordinals as for example von Neuman did when
> > > > > > he was a youngster:
> > > > > >
> > > > > > This motivates the **standard** definition, suggested by John von
> > > > > > Neumann at the age of 19, now called definition of von Neumann
> > > > > > ordinals: "each ordinal is the well-ordered set of all smaller ordinals."
> > > > > > https://en.wikipedia.org/wiki/Ordinal_number#Von_Neumann_definition_of_ordinals
> > > > > >
> > > > > > Then you have an easy canonical member inside set theory
> > > > > > for every order type. In DC Proof for every arbitrary function f
> > > > > > that is Dedekind infinite, there is some Peano structure,
> > > > > >
> > > > > > which one is canonical?
> > > > > There is nothing 'canonical' about the von Neumann representation
> > > > >
> > > > > https://en.wikipedia.org/wiki/Benacerraf%27s_identification_problem

Re: DC Proof is the biggest teaching mistake

<76bcc143-5d90-4260-a7a1-d4464d2fed21n@googlegroups.com>

  copy mid

https://www.novabbs.com/tech/article-flat.php?id=88220&group=sci.math#88220

  copy link   Newsgroups: sci.math
X-Received: by 2002:ac8:5985:: with SMTP id e5mr8637998qte.58.1642182402096;
Fri, 14 Jan 2022 09:46:42 -0800 (PST)
X-Received: by 2002:a25:804d:: with SMTP id a13mr10380680ybn.177.1642182401886;
Fri, 14 Jan 2022 09:46:41 -0800 (PST)
Path: i2pn2.org!i2pn.org!weretis.net!feeder6.news.weretis.net!news.misty.com!border2.nntp.dca1.giganews.com!border1.nntp.dca1.giganews.com!nntp.giganews.com!news-out.google.com!nntp.google.com!postnews.google.com!google-groups.googlegroups.com!not-for-mail
Newsgroups: sci.math
Date: Fri, 14 Jan 2022 09:46:41 -0800 (PST)
In-Reply-To: <ae89c82f-eb08-467f-aeaa-603ca99b5137n@googlegroups.com>
Injection-Info: google-groups.googlegroups.com; posting-host=163.182.226.42; posting-account=OWfgwwgAAADQpH2XgMDMe2wuQ7OFPXlE
NNTP-Posting-Host: 163.182.226.42
References: <bb7e1dba-3985-4555-8cf9-5c5421b69769n@googlegroups.com>
<sqvgk7$hqsh$1@solani.org> <e1dea604-6118-42a2-bc65-d1a1ffb75eecn@googlegroups.com>
<sqvhbb$hr88$2@solani.org> <411e3e45-29c7-4920-b775-0e0ef4e99f61n@googlegroups.com>
<89606698-34e1-46df-92e9-31abee426e10n@googlegroups.com> <4942e097-fe00-43ea-8d3e-24f3c6786b5dn@googlegroups.com>
<e2d289cc-e57e-4054-9544-6ae07f5bb585n@googlegroups.com> <b328f179-2765-47a4-9bec-53b3082fd847n@googlegroups.com>
<768dcb1f-80c4-457d-ac94-c89366b99ebfn@googlegroups.com> <77773d8c-4025-424c-8600-fc7ad6a922cen@googlegroups.com>
<354cd029-6017-4280-90ec-7c337be97bedn@googlegroups.com> <532b5c13-6122-4bef-a4ed-003df18cbdacn@googlegroups.com>
<4755f36f-9325-4326-b427-e25094c769dcn@googlegroups.com> <1bd61d43-8e19-4041-a050-89fb942262e6n@googlegroups.com>
<5557b6ad-6176-4dfe-81a6-758fd18813c4n@googlegroups.com> <52cc92cf-377b-4437-8d55-e933d4fba462n@googlegroups.com>
<1c2b577c-9b70-4479-ba85-c631f74079fdn@googlegroups.com> <2e6facbd-4202-4271-a8ea-2a2d1567aac8n@googlegroups.com>
<a75daafb-6adb-4d27-8650-09bcf995928fn@googlegroups.com> <8e3ad0b8-9951-4888-9593-4de90bba23d6n@googlegroups.com>
<173a8efd-28f8-4ec0-9a53-546d4bd4867an@googlegroups.com> <a64af915-4b00-4c8a-8249-47eb9b73147en@googlegroups.com>
<f097eeb2-9f31-4e0e-be3d-e4d370797c54n@googlegroups.com> <dc0b3a5a-1fa0-4729-9095-3545850cd5dcn@googlegroups.com>
<0b1bd903-87a5-4d36-a73a-ec0193b9a7e8n@googlegroups.com> <e24fc87c-feff-4493-81ca-060c9f676ec1n@googlegroups.com>
<05129406-d817-46bc-8935-ce5dbcd21a38n@googlegroups.com> <442fd60e-d3a2-4531-bb10-d2ac65552d82n@googlegroups.com>
<1304bb87-ebce-459e-a57f-e6bdf7dcebd8n@googlegroups.com> <a2df4b9b-322b-4f2e-a68f-04646fc94209n@googlegroups.com>
<ca406e7f-dc3e-4ee3-ad55-adedc315d698n@googlegroups.com> <86adee40-88d9-40df-acf6-5ed92b3dccfbn@googlegroups.com>
<20d4282a-6766-4089-ab1e-df4850c49733n@googlegroups.com> <f3fbe20c-797b-4b60-ad86-012f2d5d47f2n@googlegroups.com>
<615b2804-e5d9-4ab7-ae5e-e86c332b55e3n@googlegroups.com> <64d64634-f29e-42e6-a0cb-5230ae71afaan@googlegroups.com>
<b29ef605-a93e-49de-bec1-a6bdcc88f089n@googlegroups.com> <3b8e2323-473d-4031-8e23-9f92c5f3ea2fn@googlegroups.com>
<c1465a3d-f483-4088-ab45-ddca5e898a15n@googlegroups.com> <b41d5082-06db-4dfd-9d36-126683cba7fcn@googlegroups.com>
<95c67044-56f2-44f3-802c-575624de0281n@googlegroups.com> <bfad1477-322f-4e59-a9b2-9bd6ef2c9421n@googlegroups.com>
<0760211b-e4d4-4de3-8fd5-16c50c1b9251n@googlegroups.com> <a6120809-9a33-42fe-9590-4906f14345ddn@googlegroups.com>
<84f32020-acd7-481f-ac56-72755cce1a8an@googlegroups.com> <46991c1f-43b2-4806-b6fe-994faf80011dn@googlegroups.com>
<2f9be317-39b8-4516-992f-0d6b36bd324fn@googlegroups.com> <9149ed4f-04f6-4e88-b46f-f2da25f1c7aen@googlegroups.com>
<8eeea3c8-b5b1-4362-a4e9-bf005590a611n@googlegroups.com> <0b1410c9-f30b-4244-a326-c914e3f8825bn@googlegroups.com>
<6f5c7bca-ad0f-4daa-9424-91ae97e0fe1bn@googlegroups.com> <a56251eb-9116-4706-b692-8a6ebc2cfc13n@googlegroups.com>
<3a68a2e7-4251-4b07-ba93-2b873afd5d47n@googlegroups.com> <fa233e1d-7f3c-452e-8130-51ac7a9cde55n@googlegroups.com>
<a42ad378-d7fb-48e1-8e94-4aab3f8f6ecan@googlegroups.com> <c807383c-f994-4335-aef1-1149d9a6aaccn@googlegroups.com>
<855ecc7f-8e21-462d-bbb3-68788eda8529n@googlegroups.com> <ae89c82f-eb08-467f-aeaa-603ca99b5137n@googlegroups.com>
User-Agent: G2/1.0
MIME-Version: 1.0
Message-ID: <76bcc143-5d90-4260-a7a1-d4464d2fed21n@googlegroups.com>
Subject: Re: DC Proof is the biggest teaching mistake
From: Dan_Chri...@sympatico.ca (Dan Christensen)
Injection-Date: Fri, 14 Jan 2022 17:46:42 +0000
Content-Type: text/plain; charset="UTF-8"
Content-Transfer-Encoding: quoted-printable
Lines: 15
 by: Dan Christensen - Fri, 14 Jan 2022 17:46 UTC

On Friday, January 14, 2022 at 12:30:21 PM UTC-5, Mostowski Collapse wrote:

> ... there is still no proof
> of existence of some natural numbers in DC Proof.

In DC Proof, there is no assumption of a non-empty universe. None of its axioms of set theory postulate the existence of any set or other object, not even the empty set. So, there are no existential theorems in DC Proof, i.e. no theorems of the form EXIST(a):P(a). Peano's Axioms, which effectively define the natural numbers, are self-evident and very intuitive and can be introduced by the user at the beginning of any proof.

Dan

Download my DC Proof 2.0 freeware at http://www.dcproof.com
Visit my Math Blog at http://www.dcproof.wordpress.com

Re: DC Proof is the biggest teaching mistake

<28e6e996-5fdd-447a-9c60-93171380d58dn@googlegroups.com>

  copy mid

https://www.novabbs.com/tech/article-flat.php?id=88421&group=sci.math#88421

  copy link   Newsgroups: sci.math
X-Received: by 2002:a05:622a:411:: with SMTP id n17mr14966337qtx.439.1642363564766;
Sun, 16 Jan 2022 12:06:04 -0800 (PST)
X-Received: by 2002:a25:4cc5:: with SMTP id z188mr22936271yba.248.1642363564564;
Sun, 16 Jan 2022 12:06:04 -0800 (PST)
Path: i2pn2.org!i2pn.org!weretis.net!feeder6.news.weretis.net!1.us.feeder.erje.net!2.us.feeder.erje.net!feeder.erje.net!border1.nntp.dca1.giganews.com!nntp.giganews.com!news-out.google.com!nntp.google.com!postnews.google.com!google-groups.googlegroups.com!not-for-mail
Newsgroups: sci.math
Date: Sun, 16 Jan 2022 12:06:04 -0800 (PST)
In-Reply-To: <76bcc143-5d90-4260-a7a1-d4464d2fed21n@googlegroups.com>
Injection-Info: google-groups.googlegroups.com; posting-host=77.57.53.70; posting-account=UjEXBwoAAAAOk5fiB8WdHvZddFg9nJ9r
NNTP-Posting-Host: 77.57.53.70
References: <bb7e1dba-3985-4555-8cf9-5c5421b69769n@googlegroups.com>
<sqvgk7$hqsh$1@solani.org> <e1dea604-6118-42a2-bc65-d1a1ffb75eecn@googlegroups.com>
<sqvhbb$hr88$2@solani.org> <411e3e45-29c7-4920-b775-0e0ef4e99f61n@googlegroups.com>
<89606698-34e1-46df-92e9-31abee426e10n@googlegroups.com> <4942e097-fe00-43ea-8d3e-24f3c6786b5dn@googlegroups.com>
<e2d289cc-e57e-4054-9544-6ae07f5bb585n@googlegroups.com> <b328f179-2765-47a4-9bec-53b3082fd847n@googlegroups.com>
<768dcb1f-80c4-457d-ac94-c89366b99ebfn@googlegroups.com> <77773d8c-4025-424c-8600-fc7ad6a922cen@googlegroups.com>
<354cd029-6017-4280-90ec-7c337be97bedn@googlegroups.com> <532b5c13-6122-4bef-a4ed-003df18cbdacn@googlegroups.com>
<4755f36f-9325-4326-b427-e25094c769dcn@googlegroups.com> <1bd61d43-8e19-4041-a050-89fb942262e6n@googlegroups.com>
<5557b6ad-6176-4dfe-81a6-758fd18813c4n@googlegroups.com> <52cc92cf-377b-4437-8d55-e933d4fba462n@googlegroups.com>
<1c2b577c-9b70-4479-ba85-c631f74079fdn@googlegroups.com> <2e6facbd-4202-4271-a8ea-2a2d1567aac8n@googlegroups.com>
<a75daafb-6adb-4d27-8650-09bcf995928fn@googlegroups.com> <8e3ad0b8-9951-4888-9593-4de90bba23d6n@googlegroups.com>
<173a8efd-28f8-4ec0-9a53-546d4bd4867an@googlegroups.com> <a64af915-4b00-4c8a-8249-47eb9b73147en@googlegroups.com>
<f097eeb2-9f31-4e0e-be3d-e4d370797c54n@googlegroups.com> <dc0b3a5a-1fa0-4729-9095-3545850cd5dcn@googlegroups.com>
<0b1bd903-87a5-4d36-a73a-ec0193b9a7e8n@googlegroups.com> <e24fc87c-feff-4493-81ca-060c9f676ec1n@googlegroups.com>
<05129406-d817-46bc-8935-ce5dbcd21a38n@googlegroups.com> <442fd60e-d3a2-4531-bb10-d2ac65552d82n@googlegroups.com>
<1304bb87-ebce-459e-a57f-e6bdf7dcebd8n@googlegroups.com> <a2df4b9b-322b-4f2e-a68f-04646fc94209n@googlegroups.com>
<ca406e7f-dc3e-4ee3-ad55-adedc315d698n@googlegroups.com> <86adee40-88d9-40df-acf6-5ed92b3dccfbn@googlegroups.com>
<20d4282a-6766-4089-ab1e-df4850c49733n@googlegroups.com> <f3fbe20c-797b-4b60-ad86-012f2d5d47f2n@googlegroups.com>
<615b2804-e5d9-4ab7-ae5e-e86c332b55e3n@googlegroups.com> <64d64634-f29e-42e6-a0cb-5230ae71afaan@googlegroups.com>
<b29ef605-a93e-49de-bec1-a6bdcc88f089n@googlegroups.com> <3b8e2323-473d-4031-8e23-9f92c5f3ea2fn@googlegroups.com>
<c1465a3d-f483-4088-ab45-ddca5e898a15n@googlegroups.com> <b41d5082-06db-4dfd-9d36-126683cba7fcn@googlegroups.com>
<95c67044-56f2-44f3-802c-575624de0281n@googlegroups.com> <bfad1477-322f-4e59-a9b2-9bd6ef2c9421n@googlegroups.com>
<0760211b-e4d4-4de3-8fd5-16c50c1b9251n@googlegroups.com> <a6120809-9a33-42fe-9590-4906f14345ddn@googlegroups.com>
<84f32020-acd7-481f-ac56-72755cce1a8an@googlegroups.com> <46991c1f-43b2-4806-b6fe-994faf80011dn@googlegroups.com>
<2f9be317-39b8-4516-992f-0d6b36bd324fn@googlegroups.com> <9149ed4f-04f6-4e88-b46f-f2da25f1c7aen@googlegroups.com>
<8eeea3c8-b5b1-4362-a4e9-bf005590a611n@googlegroups.com> <0b1410c9-f30b-4244-a326-c914e3f8825bn@googlegroups.com>
<6f5c7bca-ad0f-4daa-9424-91ae97e0fe1bn@googlegroups.com> <a56251eb-9116-4706-b692-8a6ebc2cfc13n@googlegroups.com>
<3a68a2e7-4251-4b07-ba93-2b873afd5d47n@googlegroups.com> <fa233e1d-7f3c-452e-8130-51ac7a9cde55n@googlegroups.com>
<a42ad378-d7fb-48e1-8e94-4aab3f8f6ecan@googlegroups.com> <c807383c-f994-4335-aef1-1149d9a6aaccn@googlegroups.com>
<855ecc7f-8e21-462d-bbb3-68788eda8529n@googlegroups.com> <ae89c82f-eb08-467f-aeaa-603ca99b5137n@googlegroups.com>
<76bcc143-5d90-4260-a7a1-d4464d2fed21n@googlegroups.com>
User-Agent: G2/1.0
MIME-Version: 1.0
Message-ID: <28e6e996-5fdd-447a-9c60-93171380d58dn@googlegroups.com>
Subject: Re: DC Proof is the biggest teaching mistake
From: burse...@gmail.com (Mostowski Collapse)
Injection-Date: Sun, 16 Jan 2022 20:06:04 +0000
Content-Type: text/plain; charset="UTF-8"
Content-Transfer-Encoding: quoted-printable
Lines: 143
 by: Mostowski Collapse - Sun, 16 Jan 2022 20:06 UTC

WHATS WRONG WITH YOU???????????????

Dan-O-Matik went bozo:
> None of its axioms of set theory postulate the existence
of any set or other object, not even the empty set.

Wrong I can use a menu item for Peano:

1 Set(n)
Axiom

2 1 ε n
Axiom

Etc...

No I can prove the empty set:

1 Set(n)
Axiom

2 1 ε n
Axiom

3 ALL(a):ALL(b):[a ε n & b ε n => a+b ε n]
Axiom

4 ALL(a):[a ε n => ~a+1=1]
Axiom

5 ALL(a):ALL(b):[a ε n & b ε n & a+1=b+1 => a=b]
Axiom

6 ALL(a):ALL(b):[a ε n & b ε n => a+(b+1)=a+b+1]
Axiom

7 ALL(a):[Set(a) & 1 ε a & ALL(b):[b ε n & b ε a => b+1 ε a] => ALL(b):[b ε n => b ε a]]
Axiom

8 EXIST(e):[Set(e) & ALL(x):[x ε e <=> x ε n & x+1=1]]
Subset, 1

9 ~EXIST(e):[Set(e) & ALL(x):~x ε e]
Premise

10 Set(b) & ALL(x):[x ε b <=> x ε n & x+1=1]
E Spec, 8

11 Set(b)
Split, 10

12 ALL(x):[x ε b <=> x ε n & x+1=1]
Split, 10

13 ~~ALL(e):~[Set(e) & ALL(x):~x ε e]
Quant, 9

14 ALL(e):~[Set(e) & ALL(x):~x ε e]
Rem DNeg, 13

15 ~[Set(b) & ALL(x):~x ε b]
U Spec, 14

16 ~~[~Set(b) | ~ALL(x):~x ε b]
DeMorgan, 15

17 ~Set(b) | ~ALL(x):~x ε b
Rem DNeg, 16

18 ~~Set(b) => ~ALL(x):~x ε b
Imply-Or, 17

19 Set(b) => ~ALL(x):~x ε b
Rem DNeg, 18

20 ~ALL(x):~x ε b
Detach, 19, 11

21 ~~EXIST(x):~~x ε b
Quant, 20

22 EXIST(x):~~x ε b
Rem DNeg, 21

23 EXIST(x):x ε b
Rem DNeg, 22

24 c ε b
E Spec, 23

25 c ε b <=> c ε n & c+1=1
U Spec, 12

26 [c ε b => c ε n & c+1=1] & [c ε n & c+1=1 => c ε b]
Iff-And, 25

27 c ε b => c ε n & c+1=1
Split, 26

28 c ε n & c+1=1 => c ε b
Split, 26

29 c ε n => ~c+1=1
U Spec, 4

30 c ε n & c+1=1
Detach, 27, 24

31 c ε n
Split, 30

32 c+1=1
Split, 30

33 ~c+1=1
Detach, 29, 31

34 c+1=1 & ~c+1=1
Join, 32, 33

35 ~~EXIST(e):[Set(e) & ALL(x):~x ε e]
Conclusion, 9

36 EXIST(e):[Set(e) & ALL(x):~x ε e]
Rem DNeg, 35

Dan Christensen schrieb am Freitag, 14. Januar 2022 um 18:46:47 UTC+1:
> On Friday, January 14, 2022 at 12:30:21 PM UTC-5, Mostowski Collapse wrote:
>
> > ... there is still no proof
> > of existence of some natural numbers in DC Proof.
> In DC Proof, there is no assumption of a non-empty universe. None of its axioms of set theory postulate the existence of any set or other object, not even the empty set. So, there are no existential theorems in DC Proof, i.e. no theorems of the form EXIST(a):P(a). Peano's Axioms, which effectively define the natural numbers, are self-evident and very intuitive and can be introduced by the user at the beginning of any proof.
> Dan
>
> Download my DC Proof 2.0 freeware at http://www.dcproof.com
> Visit my Math Blog at http://www.dcproof.wordpress.com

Re: DC Proof is the biggest teaching mistake

<562e278a-49a4-4ae8-9ddb-1be0cf98a934n@googlegroups.com>

  copy mid

https://www.novabbs.com/tech/article-flat.php?id=88426&group=sci.math#88426

  copy link   Newsgroups: sci.math
X-Received: by 2002:a05:6214:2aac:: with SMTP id js12mr15812600qvb.71.1642366162374; Sun, 16 Jan 2022 12:49:22 -0800 (PST)
X-Received: by 2002:a25:2350:: with SMTP id j77mr20390842ybj.515.1642366162218; Sun, 16 Jan 2022 12:49:22 -0800 (PST)
Path: i2pn2.org!i2pn.org!aioe.org!news.uzoreto.com!tr1.eu1.usenetexpress.com!feeder.usenetexpress.com!tr3.iad1.usenetexpress.com!border1.nntp.dca1.giganews.com!nntp.giganews.com!news-out.google.com!nntp.google.com!postnews.google.com!google-groups.googlegroups.com!not-for-mail
Newsgroups: sci.math
Date: Sun, 16 Jan 2022 12:49:22 -0800 (PST)
In-Reply-To: <28e6e996-5fdd-447a-9c60-93171380d58dn@googlegroups.com>
Injection-Info: google-groups.googlegroups.com; posting-host=163.182.226.42; posting-account=OWfgwwgAAADQpH2XgMDMe2wuQ7OFPXlE
NNTP-Posting-Host: 163.182.226.42
References: <bb7e1dba-3985-4555-8cf9-5c5421b69769n@googlegroups.com> <sqvgk7$hqsh$1@solani.org> <e1dea604-6118-42a2-bc65-d1a1ffb75eecn@googlegroups.com> <sqvhbb$hr88$2@solani.org> <411e3e45-29c7-4920-b775-0e0ef4e99f61n@googlegroups.com> <89606698-34e1-46df-92e9-31abee426e10n@googlegroups.com> <4942e097-fe00-43ea-8d3e-24f3c6786b5dn@googlegroups.com> <e2d289cc-e57e-4054-9544-6ae07f5bb585n@googlegroups.com> <b328f179-2765-47a4-9bec-53b3082fd847n@googlegroups.com> <768dcb1f-80c4-457d-ac94-c89366b99ebfn@googlegroups.com> <77773d8c-4025-424c-8600-fc7ad6a922cen@googlegroups.com> <354cd029-6017-4280-90ec-7c337be97bedn@googlegroups.com> <532b5c13-6122-4bef-a4ed-003df18cbdacn@googlegroups.com> <4755f36f-9325-4326-b427-e25094c769dcn@googlegroups.com> <1bd61d43-8e19-4041-a050-89fb942262e6n@googlegroups.com> <5557b6ad-6176-4dfe-81a6-758fd18813c4n@googlegroups.com> <52cc92cf-377b-4437-8d55-e933d4fba462n@googlegroups.com> <1c2b577c-9b70-4479-ba85-c631f74079fdn@googlegroups.com> <2e6facbd-4202-4271-a8ea-2a2d1567aac8n@googlegroups.com> <a75daafb-6adb-4d27-8650-09bcf995928fn@googlegroups.com> <8e3ad0b8-9951-4888-9593-4de90bba23d6n@googlegroups.com> <173a8efd-28f8-4ec0-9a53-546d4bd4867an@googlegroup
s.com> <a64af915-4b00-4c8a-8249-47eb9b73147en@googlegroups.com> <f097eeb2-9f31-4e0e-be3d-e4d370797c54n@googlegroups.com> <dc0b3a5a-1fa0-4729-9095-3545850cd5dcn@googlegroups.com> <0b1bd903-87a5-4d36-a73a-ec0193b9a7e8n@googlegroups.com> <e24fc87c-feff-4493-81ca-060c9f676ec1n@googlegroups.com> <05129406-d817-46bc-8935-ce5dbcd21a38n@googlegroups.com> <442fd60e-d3a2-4531-bb10-d2ac65552d82n@googlegroups.com> <1304bb87-ebce-459e-a57f-e6bdf7dcebd8n@googlegroups.com> <a2df4b9b-322b-4f2e-a68f-04646fc94209n@googlegroups.com> <ca406e7f-dc3e-4ee3-ad55-adedc315d698n@googlegroups.com> <86adee40-88d9-40df-acf6-5ed92b3dccfbn@googlegroups.com> <20d4282a-6766-4089-ab1e-df4850c49733n@googlegroups.com> <f3fbe20c-797b-4b60-ad86-012f2d5d47f2n@googlegroups.com> <615b2804-e5d9-4ab7-ae5e-e86c332b55e3n@googlegroups.com> <64d64634-f29e-42e6-a0cb-5230ae71afaan@googlegroups.com> <b29ef605-a93e-49de-bec1-a6bdcc88f089n@googlegroups.com> <3b8e2323-473d-4031-8e23-9f92c5f3ea2fn@googlegroups.com> <c1465a3d-f483-4088-ab45-ddca5e898a15n@googlegroups.com> <b41d5082-06db-4dfd-9d36-126683cba7fcn@googlegroups.com> <95c67044-56f2-44f3-802c-575624de0281n@googlegroups.com> <bfad1477-322f-4e59-a9b2-9bd6ef2c9421n@googlegroups.
com> <0760211b-e4d4-4de3-8fd5-16c50c1b9251n@googlegroups.com> <a6120809-9a33-42fe-9590-4906f14345ddn@googlegroups.com> <84f32020-acd7-481f-ac56-72755cce1a8an@googlegroups.com> <46991c1f-43b2-4806-b6fe-994faf80011dn@googlegroups.com> <2f9be317-39b8-4516-992f-0d6b36bd324fn@googlegroups.com> <9149ed4f-04f6-4e88-b46f-f2da25f1c7aen@googlegroups.com> <8eeea3c8-b5b1-4362-a4e9-bf005590a611n@googlegroups.com> <0b1410c9-f30b-4244-a326-c914e3f8825bn@googlegroups.com> <6f5c7bca-ad0f-4daa-9424-91ae97e0fe1bn@googlegroups.com> <a56251eb-9116-4706-b692-8a6ebc2cfc13n@googlegroups.com> <3a68a2e7-4251-4b07-ba93-2b873afd5d47n@googlegroups.com> <fa233e1d-7f3c-452e-8130-51ac7a9cde55n@googlegroups.com> <a42ad378-d7fb-48e1-8e94-4aab3f8f6ecan@googlegroups.com> <c807383c-f994-4335-aef1-1149d9a6aaccn@googlegroups.com> <855ecc7f-8e21-462d-bbb3-68788eda8529n@googlegroups.com> <ae89c82f-eb08-467f-aeaa-603ca99b5137n@googlegroups.com> <76bcc143-5d90-4260-a7a1-d4464d2fed21n@googlegroups.com> <28e6e996-5fdd-447a-9c60-93171380d58dn@googlegroups.com>
User-Agent: G2/1.0
MIME-Version: 1.0
Message-ID: <562e278a-49a4-4ae8-9ddb-1be0cf98a934n@googlegroups.com>
Subject: Re: DC Proof is the biggest teaching mistake
From: Dan_Chri...@sympatico.ca (Dan Christensen)
Injection-Date: Sun, 16 Jan 2022 20:49:22 +0000
Content-Type: text/plain; charset="UTF-8"
Lines: 14
 by: Dan Christensen - Sun, 16 Jan 2022 20:49 UTC

On Sunday, January 16, 2022 at 3:06:10 PM UTC-5, Mostowski Collapse wrote:
> WHATS WRONG WITH YOU???????????????
>
> Dan-O-Matik went bozo:
> > None of its axioms of set theory postulate the existence
> of any set or other object, not even the empty set.
> Wrong I can use a menu item for Peano:
>
> 1 Set(n)
> Axiom
>

This axiom is not built into the set theory of DC Proof. It would have to have been manually keyed in by the user. But you knew this, didn't you, Jan Burse? What is your problem???

Dan

Re: DC Proof is the biggest teaching mistake

<2ec718d1-1c06-4b1e-8411-0384e71721edn@googlegroups.com>

  copy mid

https://www.novabbs.com/tech/article-flat.php?id=88431&group=sci.math#88431

  copy link   Newsgroups: sci.math
X-Received: by 2002:a37:274c:: with SMTP id n73mr4884389qkn.48.1642367538658; Sun, 16 Jan 2022 13:12:18 -0800 (PST)
X-Received: by 2002:a25:46c1:: with SMTP id t184mr24697762yba.519.1642367538530; Sun, 16 Jan 2022 13:12:18 -0800 (PST)
Path: i2pn2.org!i2pn.org!aioe.org!feeder1.feed.usenet.farm!feed.usenet.farm!tr2.eu1.usenetexpress.com!feeder.usenetexpress.com!tr2.iad1.usenetexpress.com!border1.nntp.dca1.giganews.com!nntp.giganews.com!news-out.google.com!nntp.google.com!postnews.google.com!google-groups.googlegroups.com!not-for-mail
Newsgroups: sci.math
Date: Sun, 16 Jan 2022 13:12:18 -0800 (PST)
In-Reply-To: <28e6e996-5fdd-447a-9c60-93171380d58dn@googlegroups.com>
Injection-Info: google-groups.googlegroups.com; posting-host=163.182.226.42; posting-account=OWfgwwgAAADQpH2XgMDMe2wuQ7OFPXlE
NNTP-Posting-Host: 163.182.226.42
References: <bb7e1dba-3985-4555-8cf9-5c5421b69769n@googlegroups.com> <sqvgk7$hqsh$1@solani.org> <e1dea604-6118-42a2-bc65-d1a1ffb75eecn@googlegroups.com> <sqvhbb$hr88$2@solani.org> <411e3e45-29c7-4920-b775-0e0ef4e99f61n@googlegroups.com> <89606698-34e1-46df-92e9-31abee426e10n@googlegroups.com> <4942e097-fe00-43ea-8d3e-24f3c6786b5dn@googlegroups.com> <e2d289cc-e57e-4054-9544-6ae07f5bb585n@googlegroups.com> <b328f179-2765-47a4-9bec-53b3082fd847n@googlegroups.com> <768dcb1f-80c4-457d-ac94-c89366b99ebfn@googlegroups.com> <77773d8c-4025-424c-8600-fc7ad6a922cen@googlegroups.com> <354cd029-6017-4280-90ec-7c337be97bedn@googlegroups.com> <532b5c13-6122-4bef-a4ed-003df18cbdacn@googlegroups.com> <4755f36f-9325-4326-b427-e25094c769dcn@googlegroups.com> <1bd61d43-8e19-4041-a050-89fb942262e6n@googlegroups.com> <5557b6ad-6176-4dfe-81a6-758fd18813c4n@googlegroups.com> <52cc92cf-377b-4437-8d55-e933d4fba462n@googlegroups.com> <1c2b577c-9b70-4479-ba85-c631f74079fdn@googlegroups.com> <2e6facbd-4202-4271-a8ea-2a2d1567aac8n@googlegroups.com> <a75daafb-6adb-4d27-8650-09bcf995928fn@googlegroups.com> <8e3ad0b8-9951-4888-9593-4de90bba23d6n@googlegroups.com> <173a8efd-28f8-4ec0-9a53-546d4bd4867an@googlegroup
s.com> <a64af915-4b00-4c8a-8249-47eb9b73147en@googlegroups.com> <f097eeb2-9f31-4e0e-be3d-e4d370797c54n@googlegroups.com> <dc0b3a5a-1fa0-4729-9095-3545850cd5dcn@googlegroups.com> <0b1bd903-87a5-4d36-a73a-ec0193b9a7e8n@googlegroups.com> <e24fc87c-feff-4493-81ca-060c9f676ec1n@googlegroups.com> <05129406-d817-46bc-8935-ce5dbcd21a38n@googlegroups.com> <442fd60e-d3a2-4531-bb10-d2ac65552d82n@googlegroups.com> <1304bb87-ebce-459e-a57f-e6bdf7dcebd8n@googlegroups.com> <a2df4b9b-322b-4f2e-a68f-04646fc94209n@googlegroups.com> <ca406e7f-dc3e-4ee3-ad55-adedc315d698n@googlegroups.com> <86adee40-88d9-40df-acf6-5ed92b3dccfbn@googlegroups.com> <20d4282a-6766-4089-ab1e-df4850c49733n@googlegroups.com> <f3fbe20c-797b-4b60-ad86-012f2d5d47f2n@googlegroups.com> <615b2804-e5d9-4ab7-ae5e-e86c332b55e3n@googlegroups.com> <64d64634-f29e-42e6-a0cb-5230ae71afaan@googlegroups.com> <b29ef605-a93e-49de-bec1-a6bdcc88f089n@googlegroups.com> <3b8e2323-473d-4031-8e23-9f92c5f3ea2fn@googlegroups.com> <c1465a3d-f483-4088-ab45-ddca5e898a15n@googlegroups.com> <b41d5082-06db-4dfd-9d36-126683cba7fcn@googlegroups.com> <95c67044-56f2-44f3-802c-575624de0281n@googlegroups.com> <bfad1477-322f-4e59-a9b2-9bd6ef2c9421n@googlegroups.
com> <0760211b-e4d4-4de3-8fd5-16c50c1b9251n@googlegroups.com> <a6120809-9a33-42fe-9590-4906f14345ddn@googlegroups.com> <84f32020-acd7-481f-ac56-72755cce1a8an@googlegroups.com> <46991c1f-43b2-4806-b6fe-994faf80011dn@googlegroups.com> <2f9be317-39b8-4516-992f-0d6b36bd324fn@googlegroups.com> <9149ed4f-04f6-4e88-b46f-f2da25f1c7aen@googlegroups.com> <8eeea3c8-b5b1-4362-a4e9-bf005590a611n@googlegroups.com> <0b1410c9-f30b-4244-a326-c914e3f8825bn@googlegroups.com> <6f5c7bca-ad0f-4daa-9424-91ae97e0fe1bn@googlegroups.com> <a56251eb-9116-4706-b692-8a6ebc2cfc13n@googlegroups.com> <3a68a2e7-4251-4b07-ba93-2b873afd5d47n@googlegroups.com> <fa233e1d-7f3c-452e-8130-51ac7a9cde55n@googlegroups.com> <a42ad378-d7fb-48e1-8e94-4aab3f8f6ecan@googlegroups.com> <c807383c-f994-4335-aef1-1149d9a6aaccn@googlegroups.com> <855ecc7f-8e21-462d-bbb3-68788eda8529n@googlegroups.com> <ae89c82f-eb08-467f-aeaa-603ca99b5137n@googlegroups.com> <76bcc143-5d90-4260-a7a1-d4464d2fed21n@googlegroups.com> <28e6e996-5fdd-447a-9c60-93171380d58dn@googlegroups.com>
User-Agent: G2/1.0
MIME-Version: 1.0
Message-ID: <2ec718d1-1c06-4b1e-8411-0384e71721edn@googlegroups.com>
Subject: Re: DC Proof is the biggest teaching mistake
From: Dan_Chri...@sympatico.ca (Dan Christensen)
Injection-Date: Sun, 16 Jan 2022 21:12:18 +0000
Content-Type: text/plain; charset="UTF-8"
Content-Transfer-Encoding: quoted-printable
Lines: 29
 by: Dan Christensen - Sun, 16 Jan 2022 21:12 UTC

On Sunday, January 16, 2022 at 3:06:10 PM UTC-5, Mostowski Collapse wrote:
> WHATS WRONG WITH YOU???????????????
>

Do you enjoy being an asshole, Jan Burse?

> Dan-O-Matik went bozo:
> > None of its axioms of set theory postulate the existence
> of any set or other object, not even the empty set.
> Wrong I can use a menu item for Peano:
>

The set theory in DC Proof is included the Sets and Logic menus. The axioms you list here are supplementary material for the tutorial so students or teachers wouldn't have to key them in. I never use them in my own proofs here or at the blog. I only the Sets and Logic menus for writing proofs.

> 1 Set(n)
> Axiom
>
> 2 1 ε n
> Axiom
>
> Etc...
>

Dan

Re: DC Proof is the biggest teaching mistake

<1871fcbf-ee7d-46b8-b45a-653c06e12dcfn@googlegroups.com>

  copy mid

https://www.novabbs.com/tech/article-flat.php?id=88434&group=sci.math#88434

  copy link   Newsgroups: sci.math
X-Received: by 2002:a05:6214:1d21:: with SMTP id f1mr12860351qvd.100.1642368120534;
Sun, 16 Jan 2022 13:22:00 -0800 (PST)
X-Received: by 2002:a25:2350:: with SMTP id j77mr20498456ybj.515.1642368120354;
Sun, 16 Jan 2022 13:22:00 -0800 (PST)
Path: i2pn2.org!i2pn.org!eternal-september.org!reader02.eternal-september.org!border1.nntp.dca1.giganews.com!nntp.giganews.com!news-out.google.com!nntp.google.com!postnews.google.com!google-groups.googlegroups.com!not-for-mail
Newsgroups: sci.math
Date: Sun, 16 Jan 2022 13:22:00 -0800 (PST)
In-Reply-To: <28e6e996-5fdd-447a-9c60-93171380d58dn@googlegroups.com>
Injection-Info: google-groups.googlegroups.com; posting-host=77.57.53.70; posting-account=UjEXBwoAAAAOk5fiB8WdHvZddFg9nJ9r
NNTP-Posting-Host: 77.57.53.70
References: <bb7e1dba-3985-4555-8cf9-5c5421b69769n@googlegroups.com>
<sqvgk7$hqsh$1@solani.org> <e1dea604-6118-42a2-bc65-d1a1ffb75eecn@googlegroups.com>
<sqvhbb$hr88$2@solani.org> <411e3e45-29c7-4920-b775-0e0ef4e99f61n@googlegroups.com>
<89606698-34e1-46df-92e9-31abee426e10n@googlegroups.com> <4942e097-fe00-43ea-8d3e-24f3c6786b5dn@googlegroups.com>
<e2d289cc-e57e-4054-9544-6ae07f5bb585n@googlegroups.com> <b328f179-2765-47a4-9bec-53b3082fd847n@googlegroups.com>
<768dcb1f-80c4-457d-ac94-c89366b99ebfn@googlegroups.com> <77773d8c-4025-424c-8600-fc7ad6a922cen@googlegroups.com>
<354cd029-6017-4280-90ec-7c337be97bedn@googlegroups.com> <532b5c13-6122-4bef-a4ed-003df18cbdacn@googlegroups.com>
<4755f36f-9325-4326-b427-e25094c769dcn@googlegroups.com> <1bd61d43-8e19-4041-a050-89fb942262e6n@googlegroups.com>
<5557b6ad-6176-4dfe-81a6-758fd18813c4n@googlegroups.com> <52cc92cf-377b-4437-8d55-e933d4fba462n@googlegroups.com>
<1c2b577c-9b70-4479-ba85-c631f74079fdn@googlegroups.com> <2e6facbd-4202-4271-a8ea-2a2d1567aac8n@googlegroups.com>
<a75daafb-6adb-4d27-8650-09bcf995928fn@googlegroups.com> <8e3ad0b8-9951-4888-9593-4de90bba23d6n@googlegroups.com>
<173a8efd-28f8-4ec0-9a53-546d4bd4867an@googlegroups.com> <a64af915-4b00-4c8a-8249-47eb9b73147en@googlegroups.com>
<f097eeb2-9f31-4e0e-be3d-e4d370797c54n@googlegroups.com> <dc0b3a5a-1fa0-4729-9095-3545850cd5dcn@googlegroups.com>
<0b1bd903-87a5-4d36-a73a-ec0193b9a7e8n@googlegroups.com> <e24fc87c-feff-4493-81ca-060c9f676ec1n@googlegroups.com>
<05129406-d817-46bc-8935-ce5dbcd21a38n@googlegroups.com> <442fd60e-d3a2-4531-bb10-d2ac65552d82n@googlegroups.com>
<1304bb87-ebce-459e-a57f-e6bdf7dcebd8n@googlegroups.com> <a2df4b9b-322b-4f2e-a68f-04646fc94209n@googlegroups.com>
<ca406e7f-dc3e-4ee3-ad55-adedc315d698n@googlegroups.com> <86adee40-88d9-40df-acf6-5ed92b3dccfbn@googlegroups.com>
<20d4282a-6766-4089-ab1e-df4850c49733n@googlegroups.com> <f3fbe20c-797b-4b60-ad86-012f2d5d47f2n@googlegroups.com>
<615b2804-e5d9-4ab7-ae5e-e86c332b55e3n@googlegroups.com> <64d64634-f29e-42e6-a0cb-5230ae71afaan@googlegroups.com>
<b29ef605-a93e-49de-bec1-a6bdcc88f089n@googlegroups.com> <3b8e2323-473d-4031-8e23-9f92c5f3ea2fn@googlegroups.com>
<c1465a3d-f483-4088-ab45-ddca5e898a15n@googlegroups.com> <b41d5082-06db-4dfd-9d36-126683cba7fcn@googlegroups.com>
<95c67044-56f2-44f3-802c-575624de0281n@googlegroups.com> <bfad1477-322f-4e59-a9b2-9bd6ef2c9421n@googlegroups.com>
<0760211b-e4d4-4de3-8fd5-16c50c1b9251n@googlegroups.com> <a6120809-9a33-42fe-9590-4906f14345ddn@googlegroups.com>
<84f32020-acd7-481f-ac56-72755cce1a8an@googlegroups.com> <46991c1f-43b2-4806-b6fe-994faf80011dn@googlegroups.com>
<2f9be317-39b8-4516-992f-0d6b36bd324fn@googlegroups.com> <9149ed4f-04f6-4e88-b46f-f2da25f1c7aen@googlegroups.com>
<8eeea3c8-b5b1-4362-a4e9-bf005590a611n@googlegroups.com> <0b1410c9-f30b-4244-a326-c914e3f8825bn@googlegroups.com>
<6f5c7bca-ad0f-4daa-9424-91ae97e0fe1bn@googlegroups.com> <a56251eb-9116-4706-b692-8a6ebc2cfc13n@googlegroups.com>
<3a68a2e7-4251-4b07-ba93-2b873afd5d47n@googlegroups.com> <fa233e1d-7f3c-452e-8130-51ac7a9cde55n@googlegroups.com>
<a42ad378-d7fb-48e1-8e94-4aab3f8f6ecan@googlegroups.com> <c807383c-f994-4335-aef1-1149d9a6aaccn@googlegroups.com>
<855ecc7f-8e21-462d-bbb3-68788eda8529n@googlegroups.com> <ae89c82f-eb08-467f-aeaa-603ca99b5137n@googlegroups.com>
<76bcc143-5d90-4260-a7a1-d4464d2fed21n@googlegroups.com> <28e6e996-5fdd-447a-9c60-93171380d58dn@googlegroups.com>
User-Agent: G2/1.0
MIME-Version: 1.0
Message-ID: <1871fcbf-ee7d-46b8-b45a-653c06e12dcfn@googlegroups.com>
Subject: Re: DC Proof is the biggest teaching mistake
From: burse...@gmail.com (Mostowski Collapse)
Injection-Date: Sun, 16 Jan 2022 21:22:00 +0000
Content-Type: text/plain; charset="UTF-8"
Content-Transfer-Encoding: quoted-printable
Lines: 151
 by: Mostowski Collapse - Sun, 16 Jan 2022 21:22 UTC

Now I am waiting for dcproof6.exe, it will have
a new Subset axiom, that forbids that empty
sets are created. The similarity to WM are

cunning. WM has already ranted about the
empty set, so make it the scape goat of
DC Proofs failure? LoL

Mostowski Collapse schrieb am Sonntag, 16. Januar 2022 um 21:06:10 UTC+1:
> WHATS WRONG WITH YOU???????????????
>
> Dan-O-Matik went bozo:
> > None of its axioms of set theory postulate the existence
> of any set or other object, not even the empty set.
> Wrong I can use a menu item for Peano:
>
> 1 Set(n)
> Axiom
>
> 2 1 ε n
> Axiom
>
> Etc...
>
> No I can prove the empty set:
>
> 1 Set(n)
> Axiom
>
> 2 1 ε n
> Axiom
>
> 3 ALL(a):ALL(b):[a ε n & b ε n => a+b ε n]
> Axiom
>
> 4 ALL(a):[a ε n => ~a+1=1]
> Axiom
>
> 5 ALL(a):ALL(b):[a ε n & b ε n & a+1=b+1 => a=b]
> Axiom
>
> 6 ALL(a):ALL(b):[a ε n & b ε n => a+(b+1)=a+b+1]
> Axiom
>
> 7 ALL(a):[Set(a) & 1 ε a & ALL(b):[b ε n & b ε a => b+1 ε a] => ALL(b):[b ε n => b ε a]]
> Axiom
>
> 8 EXIST(e):[Set(e) & ALL(x):[x ε e <=> x ε n & x+1=1]]
> Subset, 1
>
> 9 ~EXIST(e):[Set(e) & ALL(x):~x ε e]
> Premise
>
> 10 Set(b) & ALL(x):[x ε b <=> x ε n & x+1=1]
> E Spec, 8
>
> 11 Set(b)
> Split, 10
>
> 12 ALL(x):[x ε b <=> x ε n & x+1=1]
> Split, 10
>
> 13 ~~ALL(e):~[Set(e) & ALL(x):~x ε e]
> Quant, 9
>
> 14 ALL(e):~[Set(e) & ALL(x):~x ε e]
> Rem DNeg, 13
>
> 15 ~[Set(b) & ALL(x):~x ε b]
> U Spec, 14
>
> 16 ~~[~Set(b) | ~ALL(x):~x ε b]
> DeMorgan, 15
>
> 17 ~Set(b) | ~ALL(x):~x ε b
> Rem DNeg, 16
>
> 18 ~~Set(b) => ~ALL(x):~x ε b
> Imply-Or, 17
>
> 19 Set(b) => ~ALL(x):~x ε b
> Rem DNeg, 18
>
> 20 ~ALL(x):~x ε b
> Detach, 19, 11
>
> 21 ~~EXIST(x):~~x ε b
> Quant, 20
>
> 22 EXIST(x):~~x ε b
> Rem DNeg, 21
>
> 23 EXIST(x):x ε b
> Rem DNeg, 22
>
> 24 c ε b
> E Spec, 23
>
> 25 c ε b <=> c ε n & c+1=1
> U Spec, 12
>
> 26 [c ε b => c ε n & c+1=1] & [c ε n & c+1=1 => c ε b]
> Iff-And, 25
>
> 27 c ε b => c ε n & c+1=1
> Split, 26
>
> 28 c ε n & c+1=1 => c ε b
> Split, 26
>
> 29 c ε n => ~c+1=1
> U Spec, 4
>
> 30 c ε n & c+1=1
> Detach, 27, 24
>
> 31 c ε n
> Split, 30
>
> 32 c+1=1
> Split, 30
>
> 33 ~c+1=1
> Detach, 29, 31
>
> 34 c+1=1 & ~c+1=1
> Join, 32, 33
>
> 35 ~~EXIST(e):[Set(e) & ALL(x):~x ε e]
> Conclusion, 9
>
> 36 EXIST(e):[Set(e) & ALL(x):~x ε e]
> Rem DNeg, 35
> Dan Christensen schrieb am Freitag, 14. Januar 2022 um 18:46:47 UTC+1:
> > On Friday, January 14, 2022 at 12:30:21 PM UTC-5, Mostowski Collapse wrote:
> >
> > > ... there is still no proof
> > > of existence of some natural numbers in DC Proof.
> > In DC Proof, there is no assumption of a non-empty universe. None of its axioms of set theory postulate the existence of any set or other object, not even the empty set. So, there are no existential theorems in DC Proof, i.e. no theorems of the form EXIST(a):P(a). Peano's Axioms, which effectively define the natural numbers, are self-evident and very intuitive and can be introduced by the user at the beginning of any proof.
> > Dan
> >
> > Download my DC Proof 2.0 freeware at http://www.dcproof.com
> > Visit my Math Blog at http://www.dcproof.wordpress.com

Re: DC Proof is the biggest teaching mistake

<a1ea6c84-abdd-43b5-9827-65bab84cdf84n@googlegroups.com>

  copy mid

https://www.novabbs.com/tech/article-flat.php?id=88436&group=sci.math#88436

  copy link   Newsgroups: sci.math
X-Received: by 2002:a05:6214:2269:: with SMTP id gs9mr16440582qvb.40.1642368437202;
Sun, 16 Jan 2022 13:27:17 -0800 (PST)
X-Received: by 2002:a5b:34a:: with SMTP id q10mr22228738ybp.563.1642368437022;
Sun, 16 Jan 2022 13:27:17 -0800 (PST)
Path: i2pn2.org!i2pn.org!weretis.net!feeder6.news.weretis.net!tncsrv06.tnetconsulting.net!2.us.feeder.erje.net!feeder.erje.net!border1.nntp.dca1.giganews.com!nntp.giganews.com!news-out.google.com!nntp.google.com!postnews.google.com!google-groups.googlegroups.com!not-for-mail
Newsgroups: sci.math
Date: Sun, 16 Jan 2022 13:27:16 -0800 (PST)
In-Reply-To: <1871fcbf-ee7d-46b8-b45a-653c06e12dcfn@googlegroups.com>
Injection-Info: google-groups.googlegroups.com; posting-host=77.57.53.70; posting-account=UjEXBwoAAAAOk5fiB8WdHvZddFg9nJ9r
NNTP-Posting-Host: 77.57.53.70
References: <bb7e1dba-3985-4555-8cf9-5c5421b69769n@googlegroups.com>
<sqvgk7$hqsh$1@solani.org> <e1dea604-6118-42a2-bc65-d1a1ffb75eecn@googlegroups.com>
<sqvhbb$hr88$2@solani.org> <411e3e45-29c7-4920-b775-0e0ef4e99f61n@googlegroups.com>
<89606698-34e1-46df-92e9-31abee426e10n@googlegroups.com> <4942e097-fe00-43ea-8d3e-24f3c6786b5dn@googlegroups.com>
<e2d289cc-e57e-4054-9544-6ae07f5bb585n@googlegroups.com> <b328f179-2765-47a4-9bec-53b3082fd847n@googlegroups.com>
<768dcb1f-80c4-457d-ac94-c89366b99ebfn@googlegroups.com> <77773d8c-4025-424c-8600-fc7ad6a922cen@googlegroups.com>
<354cd029-6017-4280-90ec-7c337be97bedn@googlegroups.com> <532b5c13-6122-4bef-a4ed-003df18cbdacn@googlegroups.com>
<4755f36f-9325-4326-b427-e25094c769dcn@googlegroups.com> <1bd61d43-8e19-4041-a050-89fb942262e6n@googlegroups.com>
<5557b6ad-6176-4dfe-81a6-758fd18813c4n@googlegroups.com> <52cc92cf-377b-4437-8d55-e933d4fba462n@googlegroups.com>
<1c2b577c-9b70-4479-ba85-c631f74079fdn@googlegroups.com> <2e6facbd-4202-4271-a8ea-2a2d1567aac8n@googlegroups.com>
<a75daafb-6adb-4d27-8650-09bcf995928fn@googlegroups.com> <8e3ad0b8-9951-4888-9593-4de90bba23d6n@googlegroups.com>
<173a8efd-28f8-4ec0-9a53-546d4bd4867an@googlegroups.com> <a64af915-4b00-4c8a-8249-47eb9b73147en@googlegroups.com>
<f097eeb2-9f31-4e0e-be3d-e4d370797c54n@googlegroups.com> <dc0b3a5a-1fa0-4729-9095-3545850cd5dcn@googlegroups.com>
<0b1bd903-87a5-4d36-a73a-ec0193b9a7e8n@googlegroups.com> <e24fc87c-feff-4493-81ca-060c9f676ec1n@googlegroups.com>
<05129406-d817-46bc-8935-ce5dbcd21a38n@googlegroups.com> <442fd60e-d3a2-4531-bb10-d2ac65552d82n@googlegroups.com>
<1304bb87-ebce-459e-a57f-e6bdf7dcebd8n@googlegroups.com> <a2df4b9b-322b-4f2e-a68f-04646fc94209n@googlegroups.com>
<ca406e7f-dc3e-4ee3-ad55-adedc315d698n@googlegroups.com> <86adee40-88d9-40df-acf6-5ed92b3dccfbn@googlegroups.com>
<20d4282a-6766-4089-ab1e-df4850c49733n@googlegroups.com> <f3fbe20c-797b-4b60-ad86-012f2d5d47f2n@googlegroups.com>
<615b2804-e5d9-4ab7-ae5e-e86c332b55e3n@googlegroups.com> <64d64634-f29e-42e6-a0cb-5230ae71afaan@googlegroups.com>
<b29ef605-a93e-49de-bec1-a6bdcc88f089n@googlegroups.com> <3b8e2323-473d-4031-8e23-9f92c5f3ea2fn@googlegroups.com>
<c1465a3d-f483-4088-ab45-ddca5e898a15n@googlegroups.com> <b41d5082-06db-4dfd-9d36-126683cba7fcn@googlegroups.com>
<95c67044-56f2-44f3-802c-575624de0281n@googlegroups.com> <bfad1477-322f-4e59-a9b2-9bd6ef2c9421n@googlegroups.com>
<0760211b-e4d4-4de3-8fd5-16c50c1b9251n@googlegroups.com> <a6120809-9a33-42fe-9590-4906f14345ddn@googlegroups.com>
<84f32020-acd7-481f-ac56-72755cce1a8an@googlegroups.com> <46991c1f-43b2-4806-b6fe-994faf80011dn@googlegroups.com>
<2f9be317-39b8-4516-992f-0d6b36bd324fn@googlegroups.com> <9149ed4f-04f6-4e88-b46f-f2da25f1c7aen@googlegroups.com>
<8eeea3c8-b5b1-4362-a4e9-bf005590a611n@googlegroups.com> <0b1410c9-f30b-4244-a326-c914e3f8825bn@googlegroups.com>
<6f5c7bca-ad0f-4daa-9424-91ae97e0fe1bn@googlegroups.com> <a56251eb-9116-4706-b692-8a6ebc2cfc13n@googlegroups.com>
<3a68a2e7-4251-4b07-ba93-2b873afd5d47n@googlegroups.com> <fa233e1d-7f3c-452e-8130-51ac7a9cde55n@googlegroups.com>
<a42ad378-d7fb-48e1-8e94-4aab3f8f6ecan@googlegroups.com> <c807383c-f994-4335-aef1-1149d9a6aaccn@googlegroups.com>
<855ecc7f-8e21-462d-bbb3-68788eda8529n@googlegroups.com> <ae89c82f-eb08-467f-aeaa-603ca99b5137n@googlegroups.com>
<76bcc143-5d90-4260-a7a1-d4464d2fed21n@googlegroups.com> <28e6e996-5fdd-447a-9c60-93171380d58dn@googlegroups.com>
<1871fcbf-ee7d-46b8-b45a-653c06e12dcfn@googlegroups.com>
User-Agent: G2/1.0
MIME-Version: 1.0
Message-ID: <a1ea6c84-abdd-43b5-9827-65bab84cdf84n@googlegroups.com>
Subject: Re: DC Proof is the biggest teaching mistake
From: burse...@gmail.com (Mostowski Collapse)
Injection-Date: Sun, 16 Jan 2022 21:27:17 +0000
Content-Type: text/plain; charset="UTF-8"
Content-Transfer-Encoding: quoted-printable
Lines: 187
 by: Mostowski Collapse - Sun, 16 Jan 2022 21:27 UTC

Ha Ha, cringe:

> The axioms you list here are supplementary
material for the tutorial so students or teachers
wouldn't have to key them in. I never use them in my
own proofs here or at the blog.

Well I don't need them. I only need:

> 1 Set(n)
> Axiom
>
> 2 1 ε n
> Axiom

I could also do comprehension (Subset Axiom) with:

~1 ε n & 1 ε n

But using x+1=1 was shorter. Although the proof got lengthier.

So you say there will be no empty set when I use:

1 Set(n)
Axiom

2 0 ε n
Axiom

Why? I can then do comprehension (Subset Axiom) with:

~0 ε n & 0 ε n

Got it? LoL

Mostowski Collapse schrieb am Sonntag, 16. Januar 2022 um 22:22:06 UTC+1:
> Now I am waiting for dcproof6.exe, it will have
> a new Subset axiom, that forbids that empty
> sets are created. The similarity to WM are
>
> cunning. WM has already ranted about the
> empty set, so make it the scape goat of
> DC Proofs failure? LoL
> Mostowski Collapse schrieb am Sonntag, 16. Januar 2022 um 21:06:10 UTC+1:
> > WHATS WRONG WITH YOU???????????????
> >
> > Dan-O-Matik went bozo:
> > > None of its axioms of set theory postulate the existence
> > of any set or other object, not even the empty set.
> > Wrong I can use a menu item for Peano:
> >
> > 1 Set(n)
> > Axiom
> >
> > 2 1 ε n
> > Axiom
> >
> > Etc...
> >
> > No I can prove the empty set:
> >
> > 1 Set(n)
> > Axiom
> >
> > 2 1 ε n
> > Axiom
> >
> > 3 ALL(a):ALL(b):[a ε n & b ε n => a+b ε n]
> > Axiom
> >
> > 4 ALL(a):[a ε n => ~a+1=1]
> > Axiom
> >
> > 5 ALL(a):ALL(b):[a ε n & b ε n & a+1=b+1 => a=b]
> > Axiom
> >
> > 6 ALL(a):ALL(b):[a ε n & b ε n => a+(b+1)=a+b+1]
> > Axiom
> >
> > 7 ALL(a):[Set(a) & 1 ε a & ALL(b):[b ε n & b ε a => b+1 ε a] => ALL(b):[b ε n => b ε a]]
> > Axiom
> >
> > 8 EXIST(e):[Set(e) & ALL(x):[x ε e <=> x ε n & x+1=1]]
> > Subset, 1
> >
> > 9 ~EXIST(e):[Set(e) & ALL(x):~x ε e]
> > Premise
> >
> > 10 Set(b) & ALL(x):[x ε b <=> x ε n & x+1=1]
> > E Spec, 8
> >
> > 11 Set(b)
> > Split, 10
> >
> > 12 ALL(x):[x ε b <=> x ε n & x+1=1]
> > Split, 10
> >
> > 13 ~~ALL(e):~[Set(e) & ALL(x):~x ε e]
> > Quant, 9
> >
> > 14 ALL(e):~[Set(e) & ALL(x):~x ε e]
> > Rem DNeg, 13
> >
> > 15 ~[Set(b) & ALL(x):~x ε b]
> > U Spec, 14
> >
> > 16 ~~[~Set(b) | ~ALL(x):~x ε b]
> > DeMorgan, 15
> >
> > 17 ~Set(b) | ~ALL(x):~x ε b
> > Rem DNeg, 16
> >
> > 18 ~~Set(b) => ~ALL(x):~x ε b
> > Imply-Or, 17
> >
> > 19 Set(b) => ~ALL(x):~x ε b
> > Rem DNeg, 18
> >
> > 20 ~ALL(x):~x ε b
> > Detach, 19, 11
> >
> > 21 ~~EXIST(x):~~x ε b
> > Quant, 20
> >
> > 22 EXIST(x):~~x ε b
> > Rem DNeg, 21
> >
> > 23 EXIST(x):x ε b
> > Rem DNeg, 22
> >
> > 24 c ε b
> > E Spec, 23
> >
> > 25 c ε b <=> c ε n & c+1=1
> > U Spec, 12
> >
> > 26 [c ε b => c ε n & c+1=1] & [c ε n & c+1=1 => c ε b]
> > Iff-And, 25
> >
> > 27 c ε b => c ε n & c+1=1
> > Split, 26
> >
> > 28 c ε n & c+1=1 => c ε b
> > Split, 26
> >
> > 29 c ε n => ~c+1=1
> > U Spec, 4
> >
> > 30 c ε n & c+1=1
> > Detach, 27, 24
> >
> > 31 c ε n
> > Split, 30
> >
> > 32 c+1=1
> > Split, 30
> >
> > 33 ~c+1=1
> > Detach, 29, 31
> >
> > 34 c+1=1 & ~c+1=1
> > Join, 32, 33
> >
> > 35 ~~EXIST(e):[Set(e) & ALL(x):~x ε e]
> > Conclusion, 9
> >
> > 36 EXIST(e):[Set(e) & ALL(x):~x ε e]
> > Rem DNeg, 35
> > Dan Christensen schrieb am Freitag, 14. Januar 2022 um 18:46:47 UTC+1:
> > > On Friday, January 14, 2022 at 12:30:21 PM UTC-5, Mostowski Collapse wrote:
> > >
> > > > ... there is still no proof
> > > > of existence of some natural numbers in DC Proof.
> > > In DC Proof, there is no assumption of a non-empty universe. None of its axioms of set theory postulate the existence of any set or other object, not even the empty set. So, there are no existential theorems in DC Proof, i.e. no theorems of the form EXIST(a):P(a). Peano's Axioms, which effectively define the natural numbers, are self-evident and very intuitive and can be introduced by the user at the beginning of any proof.
> > > Dan
> > >
> > > Download my DC Proof 2.0 freeware at http://www.dcproof.com
> > > Visit my Math Blog at http://www.dcproof.wordpress.com

Re: DC Proof is the biggest teaching mistake

<c7e84586-c414-4ce8-a4e3-a88132075597n@googlegroups.com>

  copy mid

https://www.novabbs.com/tech/article-flat.php?id=88439&group=sci.math#88439

  copy link   Newsgroups: sci.math
X-Received: by 2002:a05:620a:24c9:: with SMTP id m9mr12685801qkn.615.1642368909189;
Sun, 16 Jan 2022 13:35:09 -0800 (PST)
X-Received: by 2002:a25:cdc3:: with SMTP id d186mr24687485ybf.400.1642368908968;
Sun, 16 Jan 2022 13:35:08 -0800 (PST)
Path: i2pn2.org!i2pn.org!eternal-september.org!reader02.eternal-september.org!border1.nntp.dca1.giganews.com!nntp.giganews.com!news-out.google.com!nntp.google.com!postnews.google.com!google-groups.googlegroups.com!not-for-mail
Newsgroups: sci.math
Date: Sun, 16 Jan 2022 13:35:08 -0800 (PST)
In-Reply-To: <a1ea6c84-abdd-43b5-9827-65bab84cdf84n@googlegroups.com>
Injection-Info: google-groups.googlegroups.com; posting-host=77.57.53.70; posting-account=UjEXBwoAAAAOk5fiB8WdHvZddFg9nJ9r
NNTP-Posting-Host: 77.57.53.70
References: <bb7e1dba-3985-4555-8cf9-5c5421b69769n@googlegroups.com>
<sqvgk7$hqsh$1@solani.org> <e1dea604-6118-42a2-bc65-d1a1ffb75eecn@googlegroups.com>
<sqvhbb$hr88$2@solani.org> <411e3e45-29c7-4920-b775-0e0ef4e99f61n@googlegroups.com>
<89606698-34e1-46df-92e9-31abee426e10n@googlegroups.com> <4942e097-fe00-43ea-8d3e-24f3c6786b5dn@googlegroups.com>
<e2d289cc-e57e-4054-9544-6ae07f5bb585n@googlegroups.com> <b328f179-2765-47a4-9bec-53b3082fd847n@googlegroups.com>
<768dcb1f-80c4-457d-ac94-c89366b99ebfn@googlegroups.com> <77773d8c-4025-424c-8600-fc7ad6a922cen@googlegroups.com>
<354cd029-6017-4280-90ec-7c337be97bedn@googlegroups.com> <532b5c13-6122-4bef-a4ed-003df18cbdacn@googlegroups.com>
<4755f36f-9325-4326-b427-e25094c769dcn@googlegroups.com> <1bd61d43-8e19-4041-a050-89fb942262e6n@googlegroups.com>
<5557b6ad-6176-4dfe-81a6-758fd18813c4n@googlegroups.com> <52cc92cf-377b-4437-8d55-e933d4fba462n@googlegroups.com>
<1c2b577c-9b70-4479-ba85-c631f74079fdn@googlegroups.com> <2e6facbd-4202-4271-a8ea-2a2d1567aac8n@googlegroups.com>
<a75daafb-6adb-4d27-8650-09bcf995928fn@googlegroups.com> <8e3ad0b8-9951-4888-9593-4de90bba23d6n@googlegroups.com>
<173a8efd-28f8-4ec0-9a53-546d4bd4867an@googlegroups.com> <a64af915-4b00-4c8a-8249-47eb9b73147en@googlegroups.com>
<f097eeb2-9f31-4e0e-be3d-e4d370797c54n@googlegroups.com> <dc0b3a5a-1fa0-4729-9095-3545850cd5dcn@googlegroups.com>
<0b1bd903-87a5-4d36-a73a-ec0193b9a7e8n@googlegroups.com> <e24fc87c-feff-4493-81ca-060c9f676ec1n@googlegroups.com>
<05129406-d817-46bc-8935-ce5dbcd21a38n@googlegroups.com> <442fd60e-d3a2-4531-bb10-d2ac65552d82n@googlegroups.com>
<1304bb87-ebce-459e-a57f-e6bdf7dcebd8n@googlegroups.com> <a2df4b9b-322b-4f2e-a68f-04646fc94209n@googlegroups.com>
<ca406e7f-dc3e-4ee3-ad55-adedc315d698n@googlegroups.com> <86adee40-88d9-40df-acf6-5ed92b3dccfbn@googlegroups.com>
<20d4282a-6766-4089-ab1e-df4850c49733n@googlegroups.com> <f3fbe20c-797b-4b60-ad86-012f2d5d47f2n@googlegroups.com>
<615b2804-e5d9-4ab7-ae5e-e86c332b55e3n@googlegroups.com> <64d64634-f29e-42e6-a0cb-5230ae71afaan@googlegroups.com>
<b29ef605-a93e-49de-bec1-a6bdcc88f089n@googlegroups.com> <3b8e2323-473d-4031-8e23-9f92c5f3ea2fn@googlegroups.com>
<c1465a3d-f483-4088-ab45-ddca5e898a15n@googlegroups.com> <b41d5082-06db-4dfd-9d36-126683cba7fcn@googlegroups.com>
<95c67044-56f2-44f3-802c-575624de0281n@googlegroups.com> <bfad1477-322f-4e59-a9b2-9bd6ef2c9421n@googlegroups.com>
<0760211b-e4d4-4de3-8fd5-16c50c1b9251n@googlegroups.com> <a6120809-9a33-42fe-9590-4906f14345ddn@googlegroups.com>
<84f32020-acd7-481f-ac56-72755cce1a8an@googlegroups.com> <46991c1f-43b2-4806-b6fe-994faf80011dn@googlegroups.com>
<2f9be317-39b8-4516-992f-0d6b36bd324fn@googlegroups.com> <9149ed4f-04f6-4e88-b46f-f2da25f1c7aen@googlegroups.com>
<8eeea3c8-b5b1-4362-a4e9-bf005590a611n@googlegroups.com> <0b1410c9-f30b-4244-a326-c914e3f8825bn@googlegroups.com>
<6f5c7bca-ad0f-4daa-9424-91ae97e0fe1bn@googlegroups.com> <a56251eb-9116-4706-b692-8a6ebc2cfc13n@googlegroups.com>
<3a68a2e7-4251-4b07-ba93-2b873afd5d47n@googlegroups.com> <fa233e1d-7f3c-452e-8130-51ac7a9cde55n@googlegroups.com>
<a42ad378-d7fb-48e1-8e94-4aab3f8f6ecan@googlegroups.com> <c807383c-f994-4335-aef1-1149d9a6aaccn@googlegroups.com>
<855ecc7f-8e21-462d-bbb3-68788eda8529n@googlegroups.com> <ae89c82f-eb08-467f-aeaa-603ca99b5137n@googlegroups.com>
<76bcc143-5d90-4260-a7a1-d4464d2fed21n@googlegroups.com> <28e6e996-5fdd-447a-9c60-93171380d58dn@googlegroups.com>
<1871fcbf-ee7d-46b8-b45a-653c06e12dcfn@googlegroups.com> <a1ea6c84-abdd-43b5-9827-65bab84cdf84n@googlegroups.com>
User-Agent: G2/1.0
MIME-Version: 1.0
Message-ID: <c7e84586-c414-4ce8-a4e3-a88132075597n@googlegroups.com>
Subject: Re: DC Proof is the biggest teaching mistake
From: burse...@gmail.com (Mostowski Collapse)
Injection-Date: Sun, 16 Jan 2022 21:35:09 +0000
Content-Type: text/plain; charset="UTF-8"
Content-Transfer-Encoding: quoted-printable
Lines: 202
 by: Mostowski Collapse - Sun, 16 Jan 2022 21:35 UTC

Actually I need only:

1 Set(n)
Axiom

I could then use Subset axiom with the formula:

~Set(n)

This would be the shortest. Or is there a FALSUM
constant in DC Proof? On the other hand, the most
important is not the formula that is used in your

Subset axiom, but that I have at least one set, here n,
as the the set over which to perform the comprehension
in your Subset axiom.

Mostowski Collapse schrieb am Sonntag, 16. Januar 2022 um 22:27:22 UTC+1:
> Ha Ha, cringe:
> > The axioms you list here are supplementary
> material for the tutorial so students or teachers
> wouldn't have to key them in. I never use them in my
> own proofs here or at the blog.
> Well I don't need them. I only need:
> > 1 Set(n)
> > Axiom
> >
> > 2 1 ε n
> > Axiom
> I could also do comprehension (Subset Axiom) with:
>
> ~1 ε n & 1 ε n
>
> But using x+1=1 was shorter. Although the proof got lengthier.
>
> So you say there will be no empty set when I use:
>
> 1 Set(n)
> Axiom
>
> 2 0 ε n
> Axiom
>
> Why? I can then do comprehension (Subset Axiom) with:
>
> ~0 ε n & 0 ε n
>
> Got it? LoL
> Mostowski Collapse schrieb am Sonntag, 16. Januar 2022 um 22:22:06 UTC+1:
> > Now I am waiting for dcproof6.exe, it will have
> > a new Subset axiom, that forbids that empty
> > sets are created. The similarity to WM are
> >
> > cunning. WM has already ranted about the
> > empty set, so make it the scape goat of
> > DC Proofs failure? LoL
> > Mostowski Collapse schrieb am Sonntag, 16. Januar 2022 um 21:06:10 UTC+1:
> > > WHATS WRONG WITH YOU???????????????
> > >
> > > Dan-O-Matik went bozo:
> > > > None of its axioms of set theory postulate the existence
> > > of any set or other object, not even the empty set.
> > > Wrong I can use a menu item for Peano:
> > >
> > > 1 Set(n)
> > > Axiom
> > >
> > > 2 1 ε n
> > > Axiom
> > >
> > > Etc...
> > >
> > > No I can prove the empty set:
> > >
> > > 1 Set(n)
> > > Axiom
> > >
> > > 2 1 ε n
> > > Axiom
> > >
> > > 3 ALL(a):ALL(b):[a ε n & b ε n => a+b ε n]
> > > Axiom
> > >
> > > 4 ALL(a):[a ε n => ~a+1=1]
> > > Axiom
> > >
> > > 5 ALL(a):ALL(b):[a ε n & b ε n & a+1=b+1 => a=b]
> > > Axiom
> > >
> > > 6 ALL(a):ALL(b):[a ε n & b ε n => a+(b+1)=a+b+1]
> > > Axiom
> > >
> > > 7 ALL(a):[Set(a) & 1 ε a & ALL(b):[b ε n & b ε a => b+1 ε a] => ALL(b):[b ε n => b ε a]]
> > > Axiom
> > >
> > > 8 EXIST(e):[Set(e) & ALL(x):[x ε e <=> x ε n & x+1=1]]
> > > Subset, 1
> > >
> > > 9 ~EXIST(e):[Set(e) & ALL(x):~x ε e]
> > > Premise
> > >
> > > 10 Set(b) & ALL(x):[x ε b <=> x ε n & x+1=1]
> > > E Spec, 8
> > >
> > > 11 Set(b)
> > > Split, 10
> > >
> > > 12 ALL(x):[x ε b <=> x ε n & x+1=1]
> > > Split, 10
> > >
> > > 13 ~~ALL(e):~[Set(e) & ALL(x):~x ε e]
> > > Quant, 9
> > >
> > > 14 ALL(e):~[Set(e) & ALL(x):~x ε e]
> > > Rem DNeg, 13
> > >
> > > 15 ~[Set(b) & ALL(x):~x ε b]
> > > U Spec, 14
> > >
> > > 16 ~~[~Set(b) | ~ALL(x):~x ε b]
> > > DeMorgan, 15
> > >
> > > 17 ~Set(b) | ~ALL(x):~x ε b
> > > Rem DNeg, 16
> > >
> > > 18 ~~Set(b) => ~ALL(x):~x ε b
> > > Imply-Or, 17
> > >
> > > 19 Set(b) => ~ALL(x):~x ε b
> > > Rem DNeg, 18
> > >
> > > 20 ~ALL(x):~x ε b
> > > Detach, 19, 11
> > >
> > > 21 ~~EXIST(x):~~x ε b
> > > Quant, 20
> > >
> > > 22 EXIST(x):~~x ε b
> > > Rem DNeg, 21
> > >
> > > 23 EXIST(x):x ε b
> > > Rem DNeg, 22
> > >
> > > 24 c ε b
> > > E Spec, 23
> > >
> > > 25 c ε b <=> c ε n & c+1=1
> > > U Spec, 12
> > >
> > > 26 [c ε b => c ε n & c+1=1] & [c ε n & c+1=1 => c ε b]
> > > Iff-And, 25
> > >
> > > 27 c ε b => c ε n & c+1=1
> > > Split, 26
> > >
> > > 28 c ε n & c+1=1 => c ε b
> > > Split, 26
> > >
> > > 29 c ε n => ~c+1=1
> > > U Spec, 4
> > >
> > > 30 c ε n & c+1=1
> > > Detach, 27, 24
> > >
> > > 31 c ε n
> > > Split, 30
> > >
> > > 32 c+1=1
> > > Split, 30
> > >
> > > 33 ~c+1=1
> > > Detach, 29, 31
> > >
> > > 34 c+1=1 & ~c+1=1
> > > Join, 32, 33
> > >
> > > 35 ~~EXIST(e):[Set(e) & ALL(x):~x ε e]
> > > Conclusion, 9
> > >
> > > 36 EXIST(e):[Set(e) & ALL(x):~x ε e]
> > > Rem DNeg, 35
> > > Dan Christensen schrieb am Freitag, 14. Januar 2022 um 18:46:47 UTC+1:
> > > > On Friday, January 14, 2022 at 12:30:21 PM UTC-5, Mostowski Collapse wrote:
> > > >
> > > > > ... there is still no proof
> > > > > of existence of some natural numbers in DC Proof.
> > > > In DC Proof, there is no assumption of a non-empty universe. None of its axioms of set theory postulate the existence of any set or other object, not even the empty set. So, there are no existential theorems in DC Proof, i.e. no theorems of the form EXIST(a):P(a). Peano's Axioms, which effectively define the natural numbers, are self-evident and very intuitive and can be introduced by the user at the beginning of any proof.
> > > > Dan
> > > >
> > > > Download my DC Proof 2.0 freeware at http://www.dcproof.com
> > > > Visit my Math Blog at http://www.dcproof.wordpress.com

Re: DC Proof is the biggest teaching mistake

<63bd93f5-e756-491e-917f-c8efeb40b7den@googlegroups.com>

  copy mid

https://www.novabbs.com/tech/article-flat.php?id=88725&group=sci.math#88725

  copy link   Newsgroups: sci.math
X-Received: by 2002:a1c:e914:: with SMTP id q20mr4550668wmc.70.1642612524612;
Wed, 19 Jan 2022 09:15:24 -0800 (PST)
X-Received: by 2002:a25:dd46:: with SMTP id u67mr41340701ybg.729.1642612523856;
Wed, 19 Jan 2022 09:15:23 -0800 (PST)
Path: i2pn2.org!i2pn.org!weretis.net!feeder6.news.weretis.net!news.misty.com!border2.nntp.dca1.giganews.com!nntp.giganews.com!news-out.google.com!nntp.google.com!postnews.google.com!google-groups.googlegroups.com!not-for-mail
Newsgroups: sci.math
Date: Wed, 19 Jan 2022 09:15:23 -0800 (PST)
In-Reply-To: <c7e84586-c414-4ce8-a4e3-a88132075597n@googlegroups.com>
Injection-Info: google-groups.googlegroups.com; posting-host=77.57.53.70; posting-account=UjEXBwoAAAAOk5fiB8WdHvZddFg9nJ9r
NNTP-Posting-Host: 77.57.53.70
References: <bb7e1dba-3985-4555-8cf9-5c5421b69769n@googlegroups.com>
<sqvgk7$hqsh$1@solani.org> <e1dea604-6118-42a2-bc65-d1a1ffb75eecn@googlegroups.com>
<sqvhbb$hr88$2@solani.org> <411e3e45-29c7-4920-b775-0e0ef4e99f61n@googlegroups.com>
<89606698-34e1-46df-92e9-31abee426e10n@googlegroups.com> <4942e097-fe00-43ea-8d3e-24f3c6786b5dn@googlegroups.com>
<e2d289cc-e57e-4054-9544-6ae07f5bb585n@googlegroups.com> <b328f179-2765-47a4-9bec-53b3082fd847n@googlegroups.com>
<768dcb1f-80c4-457d-ac94-c89366b99ebfn@googlegroups.com> <77773d8c-4025-424c-8600-fc7ad6a922cen@googlegroups.com>
<354cd029-6017-4280-90ec-7c337be97bedn@googlegroups.com> <532b5c13-6122-4bef-a4ed-003df18cbdacn@googlegroups.com>
<4755f36f-9325-4326-b427-e25094c769dcn@googlegroups.com> <1bd61d43-8e19-4041-a050-89fb942262e6n@googlegroups.com>
<5557b6ad-6176-4dfe-81a6-758fd18813c4n@googlegroups.com> <52cc92cf-377b-4437-8d55-e933d4fba462n@googlegroups.com>
<1c2b577c-9b70-4479-ba85-c631f74079fdn@googlegroups.com> <2e6facbd-4202-4271-a8ea-2a2d1567aac8n@googlegroups.com>
<a75daafb-6adb-4d27-8650-09bcf995928fn@googlegroups.com> <8e3ad0b8-9951-4888-9593-4de90bba23d6n@googlegroups.com>
<173a8efd-28f8-4ec0-9a53-546d4bd4867an@googlegroups.com> <a64af915-4b00-4c8a-8249-47eb9b73147en@googlegroups.com>
<f097eeb2-9f31-4e0e-be3d-e4d370797c54n@googlegroups.com> <dc0b3a5a-1fa0-4729-9095-3545850cd5dcn@googlegroups.com>
<0b1bd903-87a5-4d36-a73a-ec0193b9a7e8n@googlegroups.com> <e24fc87c-feff-4493-81ca-060c9f676ec1n@googlegroups.com>
<05129406-d817-46bc-8935-ce5dbcd21a38n@googlegroups.com> <442fd60e-d3a2-4531-bb10-d2ac65552d82n@googlegroups.com>
<1304bb87-ebce-459e-a57f-e6bdf7dcebd8n@googlegroups.com> <a2df4b9b-322b-4f2e-a68f-04646fc94209n@googlegroups.com>
<ca406e7f-dc3e-4ee3-ad55-adedc315d698n@googlegroups.com> <86adee40-88d9-40df-acf6-5ed92b3dccfbn@googlegroups.com>
<20d4282a-6766-4089-ab1e-df4850c49733n@googlegroups.com> <f3fbe20c-797b-4b60-ad86-012f2d5d47f2n@googlegroups.com>
<615b2804-e5d9-4ab7-ae5e-e86c332b55e3n@googlegroups.com> <64d64634-f29e-42e6-a0cb-5230ae71afaan@googlegroups.com>
<b29ef605-a93e-49de-bec1-a6bdcc88f089n@googlegroups.com> <3b8e2323-473d-4031-8e23-9f92c5f3ea2fn@googlegroups.com>
<c1465a3d-f483-4088-ab45-ddca5e898a15n@googlegroups.com> <b41d5082-06db-4dfd-9d36-126683cba7fcn@googlegroups.com>
<95c67044-56f2-44f3-802c-575624de0281n@googlegroups.com> <bfad1477-322f-4e59-a9b2-9bd6ef2c9421n@googlegroups.com>
<0760211b-e4d4-4de3-8fd5-16c50c1b9251n@googlegroups.com> <a6120809-9a33-42fe-9590-4906f14345ddn@googlegroups.com>
<84f32020-acd7-481f-ac56-72755cce1a8an@googlegroups.com> <46991c1f-43b2-4806-b6fe-994faf80011dn@googlegroups.com>
<2f9be317-39b8-4516-992f-0d6b36bd324fn@googlegroups.com> <9149ed4f-04f6-4e88-b46f-f2da25f1c7aen@googlegroups.com>
<8eeea3c8-b5b1-4362-a4e9-bf005590a611n@googlegroups.com> <0b1410c9-f30b-4244-a326-c914e3f8825bn@googlegroups.com>
<6f5c7bca-ad0f-4daa-9424-91ae97e0fe1bn@googlegroups.com> <a56251eb-9116-4706-b692-8a6ebc2cfc13n@googlegroups.com>
<3a68a2e7-4251-4b07-ba93-2b873afd5d47n@googlegroups.com> <fa233e1d-7f3c-452e-8130-51ac7a9cde55n@googlegroups.com>
<a42ad378-d7fb-48e1-8e94-4aab3f8f6ecan@googlegroups.com> <c807383c-f994-4335-aef1-1149d9a6aaccn@googlegroups.com>
<855ecc7f-8e21-462d-bbb3-68788eda8529n@googlegroups.com> <ae89c82f-eb08-467f-aeaa-603ca99b5137n@googlegroups.com>
<76bcc143-5d90-4260-a7a1-d4464d2fed21n@googlegroups.com> <28e6e996-5fdd-447a-9c60-93171380d58dn@googlegroups.com>
<1871fcbf-ee7d-46b8-b45a-653c06e12dcfn@googlegroups.com> <a1ea6c84-abdd-43b5-9827-65bab84cdf84n@googlegroups.com>
<c7e84586-c414-4ce8-a4e3-a88132075597n@googlegroups.com>
User-Agent: G2/1.0
MIME-Version: 1.0
Message-ID: <63bd93f5-e756-491e-917f-c8efeb40b7den@googlegroups.com>
Subject: Re: DC Proof is the biggest teaching mistake
From: burse...@gmail.com (Mostowski Collapse)
Injection-Date: Wed, 19 Jan 2022 17:15:24 +0000
Content-Type: text/plain; charset="UTF-8"
Content-Transfer-Encoding: quoted-printable
Lines: 26
 by: Mostowski Collapse - Wed, 19 Jan 2022 17:15 UTC

If you insist, that I should not use Peano with your Set Theory!!
How should I do that? I would like to do that, but your DC Proof

got broken recently. You changed the forall rule, and now
the forall rule asks for some f(x) e S. So set-less Peano does
not work anymore, I can even not prove this here anymore:

∀x∃y(y=f(x) ∧ ∀z(z=f(x) → y=z)) is valid.
https://www.umsu.de/trees/#~6x~7y%28y=f%28x%29~1~6z%28z=f%28x%29~5y=z%29%29

Its not anymore provable in DC Proof. LoL But you can check
yourself, this part of Landau was set-less, no x e N side
conditions in Landau screenshot:

MSE Question has Landau screenshot
[(e,f,g)∈d⟹(e,S(f),S(g))∈d]
Dan-O-Matiks Add Construction
https://math.stackexchange.com/a/784504/1002973

So whats going on here?

Dan Christensen schrieb am Montag, 17. Januar 2022 um 02:19:29 UTC+1:
> The axioms of set theory do NOT include Peano's Axioms. In DC Proof, the axioms of set theory are listed on the Sets menu. But you know this, don't you, Jan Burse? Why are you behaving like this, Jan Burse?

Re: DC Proof is the biggest teaching mistake

<ss9hjo$evp$1@gioia.aioe.org>

  copy mid

https://www.novabbs.com/tech/article-flat.php?id=88726&group=sci.math#88726

  copy link   Newsgroups: sci.math
Path: i2pn2.org!i2pn.org!aioe.org!tKeDShd/hwLggvz1at/JTQ.user.46.165.242.75.POSTED!not-for-mail
From: vbn...@tyut.hl (Crypto Rich)
Newsgroups: sci.math
Subject: Re: DC Proof is the biggest teaching mistake
Date: Wed, 19 Jan 2022 17:26:16 -0000 (UTC)
Organization: Aioe.org NNTP Server
Message-ID: <ss9hjo$evp$1@gioia.aioe.org>
References: <bb7e1dba-3985-4555-8cf9-5c5421b69769n@googlegroups.com>
<8eeea3c8-b5b1-4362-a4e9-bf005590a611n@googlegroups.com>
<0b1410c9-f30b-4244-a326-c914e3f8825bn@googlegroups.com>
<6f5c7bca-ad0f-4daa-9424-91ae97e0fe1bn@googlegroups.com>
<a56251eb-9116-4706-b692-8a6ebc2cfc13n@googlegroups.com>
<3a68a2e7-4251-4b07-ba93-2b873afd5d47n@googlegroups.com>
<fa233e1d-7f3c-452e-8130-51ac7a9cde55n@googlegroups.com>
<a42ad378-d7fb-48e1-8e94-4aab3f8f6ecan@googlegroups.com>
<c807383c-f994-4335-aef1-1149d9a6aaccn@googlegroups.com>
<855ecc7f-8e21-462d-bbb3-68788eda8529n@googlegroups.com>
<ae89c82f-eb08-467f-aeaa-603ca99b5137n@googlegroups.com>
<76bcc143-5d90-4260-a7a1-d4464d2fed21n@googlegroups.com>
<28e6e996-5fdd-447a-9c60-93171380d58dn@googlegroups.com>
<1871fcbf-ee7d-46b8-b45a-653c06e12dcfn@googlegroups.com>
<a1ea6c84-abdd-43b5-9827-65bab84cdf84n@googlegroups.com>
<c7e84586-c414-4ce8-a4e3-a88132075597n@googlegroups.com>
<63bd93f5-e756-491e-917f-c8efeb40b7den@googlegroups.com>
Mime-Version: 1.0
Content-Type: text/plain; charset=UTF-8
Content-Transfer-Encoding: 8bit
Injection-Info: gioia.aioe.org; logging-data="15353"; posting-host="tKeDShd/hwLggvz1at/JTQ.user.gioia.aioe.org"; mail-complaints-to="abuse@aioe.org";
User-Agent: slnr/1.0.4 (SunOS/5.10)
X-Notice: Filtered by postfilter v. 0.9.2
 by: Crypto Rich - Wed, 19 Jan 2022 17:26 UTC

Mostowski Collapse wrote:

> If you insist, that I should not use Peano with your Set Theory!!
> How should I do that? I would like to do that, but your DC Proof
>
> got broken recently. You changed the forall rule, and now the forall
> rule asks for some f(x) e S. So set-less Peano does not work anymore, I
> can even not prove this here anymore:

PRESIDENT OF EU PARLIAMENT DIES FROM RAPID ONSET OF AIDS DUE TO VACCINE...
https://www.bitchute.com/video/mdK8K1KHDIAC/

Re: DC Proof is the biggest teaching mistake

<02ecbb35-bcec-4844-bfde-aa358a077996n@googlegroups.com>

  copy mid

https://www.novabbs.com/tech/article-flat.php?id=88727&group=sci.math#88727

  copy link   Newsgroups: sci.math
X-Received: by 2002:a05:620a:e0c:: with SMTP id y12mr22084468qkm.109.1642614756966;
Wed, 19 Jan 2022 09:52:36 -0800 (PST)
X-Received: by 2002:a25:804d:: with SMTP id a13mr36889040ybn.177.1642614756782;
Wed, 19 Jan 2022 09:52:36 -0800 (PST)
Path: i2pn2.org!i2pn.org!weretis.net!feeder6.news.weretis.net!news.misty.com!border2.nntp.dca1.giganews.com!nntp.giganews.com!news-out.google.com!nntp.google.com!postnews.google.com!google-groups.googlegroups.com!not-for-mail
Newsgroups: sci.math
Date: Wed, 19 Jan 2022 09:52:36 -0800 (PST)
In-Reply-To: <a1ea6c84-abdd-43b5-9827-65bab84cdf84n@googlegroups.com>
Injection-Info: google-groups.googlegroups.com; posting-host=163.182.226.42; posting-account=OWfgwwgAAADQpH2XgMDMe2wuQ7OFPXlE
NNTP-Posting-Host: 163.182.226.42
References: <bb7e1dba-3985-4555-8cf9-5c5421b69769n@googlegroups.com>
<sqvgk7$hqsh$1@solani.org> <e1dea604-6118-42a2-bc65-d1a1ffb75eecn@googlegroups.com>
<sqvhbb$hr88$2@solani.org> <411e3e45-29c7-4920-b775-0e0ef4e99f61n@googlegroups.com>
<89606698-34e1-46df-92e9-31abee426e10n@googlegroups.com> <4942e097-fe00-43ea-8d3e-24f3c6786b5dn@googlegroups.com>
<e2d289cc-e57e-4054-9544-6ae07f5bb585n@googlegroups.com> <b328f179-2765-47a4-9bec-53b3082fd847n@googlegroups.com>
<768dcb1f-80c4-457d-ac94-c89366b99ebfn@googlegroups.com> <77773d8c-4025-424c-8600-fc7ad6a922cen@googlegroups.com>
<354cd029-6017-4280-90ec-7c337be97bedn@googlegroups.com> <532b5c13-6122-4bef-a4ed-003df18cbdacn@googlegroups.com>
<4755f36f-9325-4326-b427-e25094c769dcn@googlegroups.com> <1bd61d43-8e19-4041-a050-89fb942262e6n@googlegroups.com>
<5557b6ad-6176-4dfe-81a6-758fd18813c4n@googlegroups.com> <52cc92cf-377b-4437-8d55-e933d4fba462n@googlegroups.com>
<1c2b577c-9b70-4479-ba85-c631f74079fdn@googlegroups.com> <2e6facbd-4202-4271-a8ea-2a2d1567aac8n@googlegroups.com>
<a75daafb-6adb-4d27-8650-09bcf995928fn@googlegroups.com> <8e3ad0b8-9951-4888-9593-4de90bba23d6n@googlegroups.com>
<173a8efd-28f8-4ec0-9a53-546d4bd4867an@googlegroups.com> <a64af915-4b00-4c8a-8249-47eb9b73147en@googlegroups.com>
<f097eeb2-9f31-4e0e-be3d-e4d370797c54n@googlegroups.com> <dc0b3a5a-1fa0-4729-9095-3545850cd5dcn@googlegroups.com>
<0b1bd903-87a5-4d36-a73a-ec0193b9a7e8n@googlegroups.com> <e24fc87c-feff-4493-81ca-060c9f676ec1n@googlegroups.com>
<05129406-d817-46bc-8935-ce5dbcd21a38n@googlegroups.com> <442fd60e-d3a2-4531-bb10-d2ac65552d82n@googlegroups.com>
<1304bb87-ebce-459e-a57f-e6bdf7dcebd8n@googlegroups.com> <a2df4b9b-322b-4f2e-a68f-04646fc94209n@googlegroups.com>
<ca406e7f-dc3e-4ee3-ad55-adedc315d698n@googlegroups.com> <86adee40-88d9-40df-acf6-5ed92b3dccfbn@googlegroups.com>
<20d4282a-6766-4089-ab1e-df4850c49733n@googlegroups.com> <f3fbe20c-797b-4b60-ad86-012f2d5d47f2n@googlegroups.com>
<615b2804-e5d9-4ab7-ae5e-e86c332b55e3n@googlegroups.com> <64d64634-f29e-42e6-a0cb-5230ae71afaan@googlegroups.com>
<b29ef605-a93e-49de-bec1-a6bdcc88f089n@googlegroups.com> <3b8e2323-473d-4031-8e23-9f92c5f3ea2fn@googlegroups.com>
<c1465a3d-f483-4088-ab45-ddca5e898a15n@googlegroups.com> <b41d5082-06db-4dfd-9d36-126683cba7fcn@googlegroups.com>
<95c67044-56f2-44f3-802c-575624de0281n@googlegroups.com> <bfad1477-322f-4e59-a9b2-9bd6ef2c9421n@googlegroups.com>
<0760211b-e4d4-4de3-8fd5-16c50c1b9251n@googlegroups.com> <a6120809-9a33-42fe-9590-4906f14345ddn@googlegroups.com>
<84f32020-acd7-481f-ac56-72755cce1a8an@googlegroups.com> <46991c1f-43b2-4806-b6fe-994faf80011dn@googlegroups.com>
<2f9be317-39b8-4516-992f-0d6b36bd324fn@googlegroups.com> <9149ed4f-04f6-4e88-b46f-f2da25f1c7aen@googlegroups.com>
<8eeea3c8-b5b1-4362-a4e9-bf005590a611n@googlegroups.com> <0b1410c9-f30b-4244-a326-c914e3f8825bn@googlegroups.com>
<6f5c7bca-ad0f-4daa-9424-91ae97e0fe1bn@googlegroups.com> <a56251eb-9116-4706-b692-8a6ebc2cfc13n@googlegroups.com>
<3a68a2e7-4251-4b07-ba93-2b873afd5d47n@googlegroups.com> <fa233e1d-7f3c-452e-8130-51ac7a9cde55n@googlegroups.com>
<a42ad378-d7fb-48e1-8e94-4aab3f8f6ecan@googlegroups.com> <c807383c-f994-4335-aef1-1149d9a6aaccn@googlegroups.com>
<855ecc7f-8e21-462d-bbb3-68788eda8529n@googlegroups.com> <ae89c82f-eb08-467f-aeaa-603ca99b5137n@googlegroups.com>
<76bcc143-5d90-4260-a7a1-d4464d2fed21n@googlegroups.com> <28e6e996-5fdd-447a-9c60-93171380d58dn@googlegroups.com>
<1871fcbf-ee7d-46b8-b45a-653c06e12dcfn@googlegroups.com> <a1ea6c84-abdd-43b5-9827-65bab84cdf84n@googlegroups.com>
User-Agent: G2/1.0
MIME-Version: 1.0
Message-ID: <02ecbb35-bcec-4844-bfde-aa358a077996n@googlegroups.com>
Subject: Re: DC Proof is the biggest teaching mistake
From: Dan_Chri...@sympatico.ca (Dan Christensen)
Injection-Date: Wed, 19 Jan 2022 17:52:36 +0000
Content-Type: text/plain; charset="UTF-8"
Content-Transfer-Encoding: quoted-printable
Lines: 47
 by: Dan Christensen - Wed, 19 Jan 2022 17:52 UTC

On Sunday, January 16, 2022 at 4:27:22 PM UTC-5, Mostowski Collapse wrote:

> Well I don't need them. I only need:
> > 1 Set(n)
> > Axiom
> >
> > 2 1 ε n

You don't even need that. See below.

> > Axiom
> I could also do comprehension (Subset Axiom) with:
>
> ~1 ε n & 1 ε n
>
> But using x+1=1 was shorter. Although the proof got lengthier.
>
> So you say there will be no empty set when I use:
>
> 1 Set(n)
> Axiom
>
> 2 0 ε n
> Axiom
>
> Why? I can then do comprehension (Subset Axiom) with:
>
> ~0 ε n & 0 ε n
>

Hint: Try...

1. Set(x)
Premise

2. EXIST(a):[Set(a) & ALL(b):[b in a <=> b in x & ~b in x]]
Subset, 1

3. Set(y) & ALL(b):[b in y <=> b in x & ~b in x]
E Spec, 2

Then prove by contradiction that y is empty. I hope this helps.

Dan

Download my DC Proof 2.0 freeware at http://www.dcproof.com
Visit my Math Blog at http://www.dcproof.wordpress.com

Re: DC Proof is the biggest teaching mistake

<1f3863ad-bd33-4e5e-8234-2fbe4e8b60d3n@googlegroups.com>

  copy mid

https://www.novabbs.com/tech/article-flat.php?id=88814&group=sci.math#88814

  copy link   Newsgroups: sci.math
X-Received: by 2002:a05:622a:548:: with SMTP id m8mr166873qtx.300.1642701968541;
Thu, 20 Jan 2022 10:06:08 -0800 (PST)
X-Received: by 2002:a25:46c1:: with SMTP id t184mr286307yba.519.1642701968314;
Thu, 20 Jan 2022 10:06:08 -0800 (PST)
Path: i2pn2.org!i2pn.org!weretis.net!feeder6.news.weretis.net!1.us.feeder.erje.net!2.us.feeder.erje.net!feeder.erje.net!border1.nntp.dca1.giganews.com!nntp.giganews.com!news-out.google.com!nntp.google.com!postnews.google.com!google-groups.googlegroups.com!not-for-mail
Newsgroups: sci.math
Date: Thu, 20 Jan 2022 10:06:08 -0800 (PST)
In-Reply-To: <02ecbb35-bcec-4844-bfde-aa358a077996n@googlegroups.com>
Injection-Info: google-groups.googlegroups.com; posting-host=77.57.53.70; posting-account=UjEXBwoAAAAOk5fiB8WdHvZddFg9nJ9r
NNTP-Posting-Host: 77.57.53.70
References: <bb7e1dba-3985-4555-8cf9-5c5421b69769n@googlegroups.com>
<sqvgk7$hqsh$1@solani.org> <e1dea604-6118-42a2-bc65-d1a1ffb75eecn@googlegroups.com>
<sqvhbb$hr88$2@solani.org> <411e3e45-29c7-4920-b775-0e0ef4e99f61n@googlegroups.com>
<89606698-34e1-46df-92e9-31abee426e10n@googlegroups.com> <4942e097-fe00-43ea-8d3e-24f3c6786b5dn@googlegroups.com>
<e2d289cc-e57e-4054-9544-6ae07f5bb585n@googlegroups.com> <b328f179-2765-47a4-9bec-53b3082fd847n@googlegroups.com>
<768dcb1f-80c4-457d-ac94-c89366b99ebfn@googlegroups.com> <77773d8c-4025-424c-8600-fc7ad6a922cen@googlegroups.com>
<354cd029-6017-4280-90ec-7c337be97bedn@googlegroups.com> <532b5c13-6122-4bef-a4ed-003df18cbdacn@googlegroups.com>
<4755f36f-9325-4326-b427-e25094c769dcn@googlegroups.com> <1bd61d43-8e19-4041-a050-89fb942262e6n@googlegroups.com>
<5557b6ad-6176-4dfe-81a6-758fd18813c4n@googlegroups.com> <52cc92cf-377b-4437-8d55-e933d4fba462n@googlegroups.com>
<1c2b577c-9b70-4479-ba85-c631f74079fdn@googlegroups.com> <2e6facbd-4202-4271-a8ea-2a2d1567aac8n@googlegroups.com>
<a75daafb-6adb-4d27-8650-09bcf995928fn@googlegroups.com> <8e3ad0b8-9951-4888-9593-4de90bba23d6n@googlegroups.com>
<173a8efd-28f8-4ec0-9a53-546d4bd4867an@googlegroups.com> <a64af915-4b00-4c8a-8249-47eb9b73147en@googlegroups.com>
<f097eeb2-9f31-4e0e-be3d-e4d370797c54n@googlegroups.com> <dc0b3a5a-1fa0-4729-9095-3545850cd5dcn@googlegroups.com>
<0b1bd903-87a5-4d36-a73a-ec0193b9a7e8n@googlegroups.com> <e24fc87c-feff-4493-81ca-060c9f676ec1n@googlegroups.com>
<05129406-d817-46bc-8935-ce5dbcd21a38n@googlegroups.com> <442fd60e-d3a2-4531-bb10-d2ac65552d82n@googlegroups.com>
<1304bb87-ebce-459e-a57f-e6bdf7dcebd8n@googlegroups.com> <a2df4b9b-322b-4f2e-a68f-04646fc94209n@googlegroups.com>
<ca406e7f-dc3e-4ee3-ad55-adedc315d698n@googlegroups.com> <86adee40-88d9-40df-acf6-5ed92b3dccfbn@googlegroups.com>
<20d4282a-6766-4089-ab1e-df4850c49733n@googlegroups.com> <f3fbe20c-797b-4b60-ad86-012f2d5d47f2n@googlegroups.com>
<615b2804-e5d9-4ab7-ae5e-e86c332b55e3n@googlegroups.com> <64d64634-f29e-42e6-a0cb-5230ae71afaan@googlegroups.com>
<b29ef605-a93e-49de-bec1-a6bdcc88f089n@googlegroups.com> <3b8e2323-473d-4031-8e23-9f92c5f3ea2fn@googlegroups.com>
<c1465a3d-f483-4088-ab45-ddca5e898a15n@googlegroups.com> <b41d5082-06db-4dfd-9d36-126683cba7fcn@googlegroups.com>
<95c67044-56f2-44f3-802c-575624de0281n@googlegroups.com> <bfad1477-322f-4e59-a9b2-9bd6ef2c9421n@googlegroups.com>
<0760211b-e4d4-4de3-8fd5-16c50c1b9251n@googlegroups.com> <a6120809-9a33-42fe-9590-4906f14345ddn@googlegroups.com>
<84f32020-acd7-481f-ac56-72755cce1a8an@googlegroups.com> <46991c1f-43b2-4806-b6fe-994faf80011dn@googlegroups.com>
<2f9be317-39b8-4516-992f-0d6b36bd324fn@googlegroups.com> <9149ed4f-04f6-4e88-b46f-f2da25f1c7aen@googlegroups.com>
<8eeea3c8-b5b1-4362-a4e9-bf005590a611n@googlegroups.com> <0b1410c9-f30b-4244-a326-c914e3f8825bn@googlegroups.com>
<6f5c7bca-ad0f-4daa-9424-91ae97e0fe1bn@googlegroups.com> <a56251eb-9116-4706-b692-8a6ebc2cfc13n@googlegroups.com>
<3a68a2e7-4251-4b07-ba93-2b873afd5d47n@googlegroups.com> <fa233e1d-7f3c-452e-8130-51ac7a9cde55n@googlegroups.com>
<a42ad378-d7fb-48e1-8e94-4aab3f8f6ecan@googlegroups.com> <c807383c-f994-4335-aef1-1149d9a6aaccn@googlegroups.com>
<855ecc7f-8e21-462d-bbb3-68788eda8529n@googlegroups.com> <ae89c82f-eb08-467f-aeaa-603ca99b5137n@googlegroups.com>
<76bcc143-5d90-4260-a7a1-d4464d2fed21n@googlegroups.com> <28e6e996-5fdd-447a-9c60-93171380d58dn@googlegroups.com>
<1871fcbf-ee7d-46b8-b45a-653c06e12dcfn@googlegroups.com> <a1ea6c84-abdd-43b5-9827-65bab84cdf84n@googlegroups.com>
<02ecbb35-bcec-4844-bfde-aa358a077996n@googlegroups.com>
User-Agent: G2/1.0
MIME-Version: 1.0
Message-ID: <1f3863ad-bd33-4e5e-8234-2fbe4e8b60d3n@googlegroups.com>
Subject: Re: DC Proof is the biggest teaching mistake
From: burse...@gmail.com (Mostowski Collapse)
Injection-Date: Thu, 20 Jan 2022 18:06:08 +0000
Content-Type: text/plain; charset="UTF-8"
Content-Transfer-Encoding: quoted-printable
Lines: 33
 by: Mostowski Collapse - Thu, 20 Jan 2022 18:06 UTC

Q: How do you use DC poop to prove Landau's:

∀x∀y(x + y = y + x)

A: Not at all, it doesn't use x e N. LoL

P.S.: Here is a proof in Landau, using tree tool https://www.umsu.de/trees:

1) First prove by induction that ∀x(0 + x = x).
Base Case: 0 + 0 = 0
∀xa(x,0)=x → a(0,0)=0 is valid.
Induction Step: 0 + x = x => 0 + s(x) = s(x)
∀x∀ya(x,s(y))=s(a(x,y)) → ∀x(a(0,x)=x → a(0,s(x))=s(x)) is valid.

2) Now prove by induction that ∀x∀y(s(y) + x = s(y + x))
Base Case: ∀y(s(y) + 0 = s(y + 0))
∀xa(x,0)=x → ∀ya(s(y),0)=s(a(y,0)) is valid.
Induction Step: ∀y(s(y) + x = s(y + x)) => ∀y(s(y) + s(x) = s(y + s(x)))
∀x∀ya(x,s(y))=s(a(x,y)) → ∀x(∀ya(s(y),x)=s(a(y,x)) → ∀ya(s(y),s(x))=s(a(y,s(x)))) is valid.

3) Finally prove by induction that ∀x∀y(x + y = y + x)
Base Case: 0 + y = y + 0, using 1)
(∀xa(x,0)=x ∧ ∀xa(0,x)=x) → ∀ya(0,y)=a(y,0) is valid.
Induction Step: x + y = y + x => s(x) + y = y + s(x), using 2)
(∀x∀ya(x,s(y))=s(a(x,y)) ∧ ∀x∀ya(s(y),x)=s(a(x,y))) → ∀x(∀ya(x,y)=a(y,x) → ∀ya(s(x),y)=a(y,s(x))) is valid.

Q.E.D.

Re: DC Proof is the biggest teaching mistake

<ssc9jp$csi$1@gioia.aioe.org>

  copy mid

https://www.novabbs.com/tech/article-flat.php?id=88818&group=sci.math#88818

  copy link   Newsgroups: sci.math
Path: i2pn2.org!i2pn.org!aioe.org!JYKIVjLUsClhTTAT3dHWVA.user.46.165.242.75.POSTED!not-for-mail
From: bnb...@rty.vb (Wesi Ebbs)
Newsgroups: sci.math
Subject: Re: DC Proof is the biggest teaching mistake
Date: Thu, 20 Jan 2022 18:28:09 -0000 (UTC)
Organization: Aioe.org NNTP Server
Message-ID: <ssc9jp$csi$1@gioia.aioe.org>
References: <bb7e1dba-3985-4555-8cf9-5c5421b69769n@googlegroups.com>
<8eeea3c8-b5b1-4362-a4e9-bf005590a611n@googlegroups.com>
<0b1410c9-f30b-4244-a326-c914e3f8825bn@googlegroups.com>
<6f5c7bca-ad0f-4daa-9424-91ae97e0fe1bn@googlegroups.com>
<a56251eb-9116-4706-b692-8a6ebc2cfc13n@googlegroups.com>
<3a68a2e7-4251-4b07-ba93-2b873afd5d47n@googlegroups.com>
<fa233e1d-7f3c-452e-8130-51ac7a9cde55n@googlegroups.com>
<a42ad378-d7fb-48e1-8e94-4aab3f8f6ecan@googlegroups.com>
<c807383c-f994-4335-aef1-1149d9a6aaccn@googlegroups.com>
<855ecc7f-8e21-462d-bbb3-68788eda8529n@googlegroups.com>
<ae89c82f-eb08-467f-aeaa-603ca99b5137n@googlegroups.com>
<76bcc143-5d90-4260-a7a1-d4464d2fed21n@googlegroups.com>
<28e6e996-5fdd-447a-9c60-93171380d58dn@googlegroups.com>
<1871fcbf-ee7d-46b8-b45a-653c06e12dcfn@googlegroups.com>
<a1ea6c84-abdd-43b5-9827-65bab84cdf84n@googlegroups.com>
<02ecbb35-bcec-4844-bfde-aa358a077996n@googlegroups.com>
<1f3863ad-bd33-4e5e-8234-2fbe4e8b60d3n@googlegroups.com>
Mime-Version: 1.0
Content-Type: text/plain; charset=UTF-8
Content-Transfer-Encoding: 8bit
Injection-Info: gioia.aioe.org; logging-data="13202"; posting-host="JYKIVjLUsClhTTAT3dHWVA.user.gioia.aioe.org"; mail-complaints-to="abuse@aioe.org";
User-Agent: Mozilla/5.0 (Windows NT 11.0; Win64; x64; rv:78.0) Gecko/20100101
Thunderbird/78.7.1
X-Notice: Filtered by postfilter v. 0.9.2
 by: Wesi Ebbs - Thu, 20 Jan 2022 18:28 UTC

Mostowski Collapse wrote:

> Q: How do you use DC poop to prove Landau's: ∀x∀y(x + y = y + x)
> A: Not at all, it doesn't use x e N. LoL

why did you fake money capitalists bombed Yugoslavia, then?

Re: DC Proof is the biggest teaching mistake

<de4bd6a1-b202-47bf-b3f5-5df55eca5a37n@googlegroups.com>

  copy mid

https://www.novabbs.com/tech/article-flat.php?id=88864&group=sci.math#88864

  copy link   Newsgroups: sci.math
X-Received: by 2002:a05:620a:4556:: with SMTP id u22mr1165014qkp.631.1642726234790;
Thu, 20 Jan 2022 16:50:34 -0800 (PST)
X-Received: by 2002:a25:6f43:: with SMTP id k64mr2981912ybc.206.1642726234488;
Thu, 20 Jan 2022 16:50:34 -0800 (PST)
Path: i2pn2.org!i2pn.org!weretis.net!feeder6.news.weretis.net!news.misty.com!border2.nntp.dca1.giganews.com!nntp.giganews.com!news-out.google.com!nntp.google.com!postnews.google.com!google-groups.googlegroups.com!not-for-mail
Newsgroups: sci.math
Date: Thu, 20 Jan 2022 16:50:34 -0800 (PST)
In-Reply-To: <ss9hjo$evp$1@gioia.aioe.org>
Injection-Info: google-groups.googlegroups.com; posting-host=77.57.53.70; posting-account=UjEXBwoAAAAOk5fiB8WdHvZddFg9nJ9r
NNTP-Posting-Host: 77.57.53.70
References: <bb7e1dba-3985-4555-8cf9-5c5421b69769n@googlegroups.com>
<8eeea3c8-b5b1-4362-a4e9-bf005590a611n@googlegroups.com> <0b1410c9-f30b-4244-a326-c914e3f8825bn@googlegroups.com>
<6f5c7bca-ad0f-4daa-9424-91ae97e0fe1bn@googlegroups.com> <a56251eb-9116-4706-b692-8a6ebc2cfc13n@googlegroups.com>
<3a68a2e7-4251-4b07-ba93-2b873afd5d47n@googlegroups.com> <fa233e1d-7f3c-452e-8130-51ac7a9cde55n@googlegroups.com>
<a42ad378-d7fb-48e1-8e94-4aab3f8f6ecan@googlegroups.com> <c807383c-f994-4335-aef1-1149d9a6aaccn@googlegroups.com>
<855ecc7f-8e21-462d-bbb3-68788eda8529n@googlegroups.com> <ae89c82f-eb08-467f-aeaa-603ca99b5137n@googlegroups.com>
<76bcc143-5d90-4260-a7a1-d4464d2fed21n@googlegroups.com> <28e6e996-5fdd-447a-9c60-93171380d58dn@googlegroups.com>
<1871fcbf-ee7d-46b8-b45a-653c06e12dcfn@googlegroups.com> <a1ea6c84-abdd-43b5-9827-65bab84cdf84n@googlegroups.com>
<c7e84586-c414-4ce8-a4e3-a88132075597n@googlegroups.com> <63bd93f5-e756-491e-917f-c8efeb40b7den@googlegroups.com>
<ss9hjo$evp$1@gioia.aioe.org>
User-Agent: G2/1.0
MIME-Version: 1.0
Message-ID: <de4bd6a1-b202-47bf-b3f5-5df55eca5a37n@googlegroups.com>
Subject: Re: DC Proof is the biggest teaching mistake
From: burse...@gmail.com (Mostowski Collapse)
Injection-Date: Fri, 21 Jan 2022 00:50:34 +0000
Content-Type: text/plain; charset="UTF-8"
Content-Transfer-Encoding: quoted-printable
Lines: 22
 by: Mostowski Collapse - Fri, 21 Jan 2022 00:50 UTC

Ha Ha, Crypto Rich too much fondlig with his pedophile
pope, made is brain melt. Every goog logician knows:

human(DavidSassoli)
∀x(human(x) => mortal(x))
----------------------------------------------
mortal(DavidSassoli)

So who cares?

Crypto Rich schrieb am Mittwoch, 19. Januar 2022 um 18:26:25 UTC+1:
> Mostowski Collapse wrote:
>
> > If you insist, that I should not use Peano with your Set Theory!!
> > How should I do that? I would like to do that, but your DC Proof
> >
> > got broken recently. You changed the forall rule, and now the forall
> > rule asks for some f(x) e S. So set-less Peano does not work anymore, I
> > can even not prove this here anymore:
> PRESIDENT OF EU PARLIAMENT DIES FROM RAPID ONSET OF AIDS DUE TO VACCINE....
> https://www.bitchute.com/video/mdK8K1KHDIAC/

Re: DC Proof is the biggest teaching mistake

<3c882a6b-ac52-4cf4-a39f-10dcb209d39cn@googlegroups.com>

  copy mid

https://www.novabbs.com/tech/article-flat.php?id=88865&group=sci.math#88865

  copy link   Newsgroups: sci.math
X-Received: by 2002:a05:622a:3ce:: with SMTP id k14mr1535952qtx.465.1642726453818;
Thu, 20 Jan 2022 16:54:13 -0800 (PST)
X-Received: by 2002:a5b:ec7:: with SMTP id a7mr2723929ybs.628.1642726453645;
Thu, 20 Jan 2022 16:54:13 -0800 (PST)
Path: i2pn2.org!i2pn.org!weretis.net!feeder6.news.weretis.net!news.misty.com!border2.nntp.dca1.giganews.com!nntp.giganews.com!news-out.google.com!nntp.google.com!postnews.google.com!google-groups.googlegroups.com!not-for-mail
Newsgroups: sci.math
Date: Thu, 20 Jan 2022 16:54:13 -0800 (PST)
In-Reply-To: <de4bd6a1-b202-47bf-b3f5-5df55eca5a37n@googlegroups.com>
Injection-Info: google-groups.googlegroups.com; posting-host=77.57.53.70; posting-account=UjEXBwoAAAAOk5fiB8WdHvZddFg9nJ9r
NNTP-Posting-Host: 77.57.53.70
References: <bb7e1dba-3985-4555-8cf9-5c5421b69769n@googlegroups.com>
<8eeea3c8-b5b1-4362-a4e9-bf005590a611n@googlegroups.com> <0b1410c9-f30b-4244-a326-c914e3f8825bn@googlegroups.com>
<6f5c7bca-ad0f-4daa-9424-91ae97e0fe1bn@googlegroups.com> <a56251eb-9116-4706-b692-8a6ebc2cfc13n@googlegroups.com>
<3a68a2e7-4251-4b07-ba93-2b873afd5d47n@googlegroups.com> <fa233e1d-7f3c-452e-8130-51ac7a9cde55n@googlegroups.com>
<a42ad378-d7fb-48e1-8e94-4aab3f8f6ecan@googlegroups.com> <c807383c-f994-4335-aef1-1149d9a6aaccn@googlegroups.com>
<855ecc7f-8e21-462d-bbb3-68788eda8529n@googlegroups.com> <ae89c82f-eb08-467f-aeaa-603ca99b5137n@googlegroups.com>
<76bcc143-5d90-4260-a7a1-d4464d2fed21n@googlegroups.com> <28e6e996-5fdd-447a-9c60-93171380d58dn@googlegroups.com>
<1871fcbf-ee7d-46b8-b45a-653c06e12dcfn@googlegroups.com> <a1ea6c84-abdd-43b5-9827-65bab84cdf84n@googlegroups.com>
<c7e84586-c414-4ce8-a4e3-a88132075597n@googlegroups.com> <63bd93f5-e756-491e-917f-c8efeb40b7den@googlegroups.com>
<ss9hjo$evp$1@gioia.aioe.org> <de4bd6a1-b202-47bf-b3f5-5df55eca5a37n@googlegroups.com>
User-Agent: G2/1.0
MIME-Version: 1.0
Message-ID: <3c882a6b-ac52-4cf4-a39f-10dcb209d39cn@googlegroups.com>
Subject: Re: DC Proof is the biggest teaching mistake
From: burse...@gmail.com (Mostowski Collapse)
Injection-Date: Fri, 21 Jan 2022 00:54:13 +0000
Content-Type: text/plain; charset="UTF-8"
Content-Transfer-Encoding: quoted-printable
Lines: 32
 by: Mostowski Collapse - Fri, 21 Jan 2022 00:54 UTC

Only exception is in DC poop, we cannot prove:

human(VicePresident(EuropeanParliament)))
∀x(human(x) => mortal(x))
----------------------------------------------
mortal(VicePresident(EuropeanParliament)))

LoL

Mostowski Collapse schrieb am Freitag, 21. Januar 2022 um 01:50:40 UTC+1:
> Ha Ha, Crypto Rich too much fondlig with his pedophile
> pope, made is brain melt. Every goog logician knows:
>
> human(DavidSassoli)
> ∀x(human(x) => mortal(x))
> ----------------------------------------------
> mortal(DavidSassoli)
>
> So who cares?
> Crypto Rich schrieb am Mittwoch, 19. Januar 2022 um 18:26:25 UTC+1:
> > Mostowski Collapse wrote:
> >
> > > If you insist, that I should not use Peano with your Set Theory!!
> > > How should I do that? I would like to do that, but your DC Proof
> > >
> > > got broken recently. You changed the forall rule, and now the forall
> > > rule asks for some f(x) e S. So set-less Peano does not work anymore, I
> > > can even not prove this here anymore:
> > PRESIDENT OF EU PARLIAMENT DIES FROM RAPID ONSET OF AIDS DUE TO VACCINE....
> > https://www.bitchute.com/video/mdK8K1KHDIAC/

Re: DC Proof is the biggest teaching mistake

<sseter$1eiu7$2@solani.org>

  copy mid

https://www.novabbs.com/tech/article-flat.php?id=88929&group=sci.math#88929

  copy link   Newsgroups: sci.math
Path: i2pn2.org!i2pn.org!weretis.net!feeder8.news.weretis.net!reader5.news.weretis.net!news.solani.org!.POSTED!not-for-mail
From: janbu...@fastmail.fm (Mostowski Collapse)
Newsgroups: sci.math
Subject: Re: DC Proof is the biggest teaching mistake
Date: Fri, 21 Jan 2022 19:19:04 +0100
Message-ID: <sseter$1eiu7$2@solani.org>
References: <bb7e1dba-3985-4555-8cf9-5c5421b69769n@googlegroups.com>
Mime-Version: 1.0
Content-Type: text/plain; charset=UTF-8; format=flowed
Content-Transfer-Encoding: 8bit
Injection-Date: Fri, 21 Jan 2022 18:19:07 -0000 (UTC)
Injection-Info: solani.org;
logging-data="1526727"; mail-complaints-to="abuse@news.solani.org"
User-Agent: Mozilla/5.0 (Macintosh; Intel Mac OS X 10.16; rv:68.0)
Gecko/20100101 Firefox/68.0 SeaMonkey/2.53.10.2
Cancel-Lock: sha1:1/WHX4PvaoiFNnD0AfPs0DtCvOw=
X-User-ID: eJwNyscBwCAMA8CVXCSXcQyE/Ucg9z56aOxEMMDLq+msWbbasCmUKHztLpE1XYHE/Sdx2hy5xXhaB9q6xqkPC/0TWw==
In-Reply-To: <bb7e1dba-3985-4555-8cf9-5c5421b69769n@googlegroups.com>
 by: Mostowski Collapse - Fri, 21 Jan 2022 18:19 UTC

Its rather the other way around. You proved too
much. Please read Landau carefully. He only said:

"To every pair of numbers x, y, we may assign
exactly one way a natural number"

This can be cast as:

∀x ∀y ∃!z (x + y = z)

Which translates to:

∀x ∀y ∃z (x + y = z)
∀x ∀y ∀z ∀t (x + y = z & x + y = t => z = t)

But this trivially valid in FOL=:

∀x∀y∃za(x,y)=z is valid.
https://www.umsu.de/trees/#~6x~6y~7z%28a%28x,y%29=z%29

∀x∀y∀z∀t((a(x,y)=z ∧ a(x,y)=t) → z=t) is valid.
https://www.umsu.de/trees/#~6x~6y~6z~6t%28a%28x,y%29=z~1a%28x,y%29=t~5z=t%29

Re: DC Proof is the biggest teaching mistake

<sseth8$1eiu7$3@solani.org>

  copy mid

https://www.novabbs.com/tech/article-flat.php?id=88930&group=sci.math#88930

  copy link   Newsgroups: sci.math
Path: i2pn2.org!i2pn.org!weretis.net!feeder8.news.weretis.net!reader5.news.weretis.net!news.solani.org!.POSTED!not-for-mail
From: janbu...@fastmail.fm (Mostowski Collapse)
Newsgroups: sci.math
Subject: Re: DC Proof is the biggest teaching mistake
Date: Fri, 21 Jan 2022 19:20:21 +0100
Message-ID: <sseth8$1eiu7$3@solani.org>
References: <bb7e1dba-3985-4555-8cf9-5c5421b69769n@googlegroups.com>
<sseter$1eiu7$2@solani.org>
Mime-Version: 1.0
Content-Type: text/plain; charset=UTF-8; format=flowed
Content-Transfer-Encoding: 8bit
Injection-Date: Fri, 21 Jan 2022 18:20:24 -0000 (UTC)
Injection-Info: solani.org;
logging-data="1526727"; mail-complaints-to="abuse@news.solani.org"
User-Agent: Mozilla/5.0 (Macintosh; Intel Mac OS X 10.16; rv:68.0)
Gecko/20100101 Firefox/68.0 SeaMonkey/2.53.10.2
Cancel-Lock: sha1:BolPf3nn8KQcosjW3sfJcbZUcTc=
X-User-ID: eJwNxMEBwCAIA8CVQCDIOGhk/xHae1wYFDcdAY+J+V8IO5fyGMVUl7cJ1ex+PNwIL87xDeGU6VurUpRl3TIfTF8VZg==
In-Reply-To: <sseter$1eiu7$2@solani.org>
 by: Mostowski Collapse - Fri, 21 Jan 2022 18:20 UTC

If you want to interpret Landaus "assign" differently,
in the sense of "by finite means", your proof does nothing.
You then need to go into recursion theory, and for

example show that a certain operator Φ is not only
monotonic, but also continous, and then the fixpoint
can be computed by finite union, no hierarchy jumps:

lfp(Φ) = union_n e N Φ^n

But you don't prove anything like this.

Mostowski Collapse schrieb:
> Its rather the other way around. You proved too
> much. Please read Landau carefully. He only said:
>
> "To every pair of numbers x, y, we may assign
> exactly one way a natural number"
>
> This can be cast as:
>
> ∀x ∀y ∃!z (x + y = z)
>
> Which translates to:
>
> ∀x ∀y ∃z (x + y = z)
> ∀x ∀y ∀z ∀t (x + y = z & x + y = t => z = t)
>
> But this trivially valid in FOL=:
>
> ∀x∀y∃za(x,y)=z is valid.
> https://www.umsu.de/trees/#~6x~6y~7z%28a%28x,y%29=z%29
>
> ∀x∀y∀z∀t((a(x,y)=z ∧ a(x,y)=t) → z=t) is valid.
> https://www.umsu.de/trees/#~6x~6y~6z~6t%28a%28x,y%29=z~1a%28x,y%29=t~5z=t%29
>

Re: DC Proof is the biggest teaching mistake

<sseu1i$1ejgc$1@solani.org>

  copy mid

https://www.novabbs.com/tech/article-flat.php?id=88931&group=sci.math#88931

  copy link   Newsgroups: sci.math
Path: i2pn2.org!i2pn.org!weretis.net!feeder8.news.weretis.net!reader5.news.weretis.net!news.solani.org!.POSTED!not-for-mail
From: janbu...@fastmail.fm (Mostowski Collapse)
Newsgroups: sci.math
Subject: Re: DC Proof is the biggest teaching mistake
Date: Fri, 21 Jan 2022 19:29:04 +0100
Message-ID: <sseu1i$1ejgc$1@solani.org>
References: <bb7e1dba-3985-4555-8cf9-5c5421b69769n@googlegroups.com>
<sseter$1eiu7$2@solani.org> <sseth8$1eiu7$3@solani.org>
Mime-Version: 1.0
Content-Type: text/plain; charset=UTF-8; format=flowed
Content-Transfer-Encoding: 8bit
Injection-Date: Fri, 21 Jan 2022 18:29:06 -0000 (UTC)
Injection-Info: solani.org;
logging-data="1527308"; mail-complaints-to="abuse@news.solani.org"
User-Agent: Mozilla/5.0 (Macintosh; Intel Mac OS X 10.16; rv:68.0)
Gecko/20100101 Firefox/68.0 SeaMonkey/2.53.10.2
Cancel-Lock: sha1:Xiy76M7/z/OzDcvms+MnpYLpBUM=
In-Reply-To: <sseth8$1eiu7$3@solani.org>
X-User-ID: eJwFwQcBACAIBMBKCrLisL5/BO+E9WrbU9EnEOApgihqJr28JZRD95A1DKnGudFjk7QhDgovXpYDvnbqfmVNFbQ=
 by: Mostowski Collapse - Fri, 21 Jan 2022 18:29 UTC

The problem is, math stackexchange is not the
right place to learn anything mathematics or
anything computer science.

Just imagine that you Dan-O-Matik, the anti-
troll that is himself the biggest crank has
access to math stackexchange, then

you can imagine how cranks begat each other
on math stackexchange with utter nonsense.
The Landau question is a typical example,

and answer says:

"Whenever you define something, it's essential
to prove that the thing being defined both
exists and is unique. Otherwise the definition
is meaningless."
https://math.stackexchange.com/a/783951/1002973

The technical term for such a definition,
which is unique is "welldefined". It is not
meaningless. For example I can define:

def(X) :<=> X=1 v X=2.

So what, its not meaningless. I have accused
stackoverflow of bitrot, since it doesn't allow
deletion, and hence it is often not in sync

with actual releases of software on the market.
But math stackexchange can be accused of not
being in sync with anything real mathematics

or computer science. Its kind of a bloated rot
of clown nonsense, extremly remote from the
question that are dealt with in math and cs.

Mostowski Collapse schrieb:
> If you want to interpret Landaus "assign" differently,
> in  the sense of "by finite means", your proof does nothing.
> You then need to go into recursion theory, and for
>
> example show that a certain operator Φ is not only
> monotonic, but also continous, and then the fixpoint
> can be computed by finite union, no hierarchy jumps:
>
> lfp(Φ) = union_n e N Φ^n
>
> But you don't prove anything like this.
>
> Mostowski Collapse schrieb:
>> Its rather the other way around. You proved too
>> much. Please read Landau carefully. He only said:
>>
>> "To every pair of numbers x, y, we may assign
>> exactly one way a natural number"
>>
>> This can be cast as:
>>
>> ∀x ∀y ∃!z (x + y = z)
>>
>> Which translates to:
>>
>> ∀x ∀y ∃z (x + y = z)
>> ∀x ∀y ∀z ∀t (x + y = z & x + y = t => z = t)
>>
>> But this trivially valid in FOL=:
>>
>> ∀x∀y∃za(x,y)=z is valid.
>> https://www.umsu.de/trees/#~6x~6y~7z%28a%28x,y%29=z%29
>>
>> ∀x∀y∀z∀t((a(x,y)=z ∧ a(x,y)=t) → z=t) is valid.
>> https://www.umsu.de/trees/#~6x~6y~6z~6t%28a%28x,y%29=z~1a%28x,y%29=t~5z=t%29
>>
>

Re: DC Proof is the biggest teaching mistake

<7141d086-c610-45b2-ba9e-11c4f15048e6n@googlegroups.com>

  copy mid

https://www.novabbs.com/tech/article-flat.php?id=88934&group=sci.math#88934

  copy link   Newsgroups: sci.math
X-Received: by 2002:ac8:7f88:: with SMTP id z8mr4526015qtj.396.1642791417451;
Fri, 21 Jan 2022 10:56:57 -0800 (PST)
X-Received: by 2002:a5b:34a:: with SMTP id q10mr7762357ybp.563.1642791417304;
Fri, 21 Jan 2022 10:56:57 -0800 (PST)
Path: i2pn2.org!i2pn.org!weretis.net!feeder6.news.weretis.net!news.misty.com!border2.nntp.dca1.giganews.com!nntp.giganews.com!news-out.google.com!nntp.google.com!postnews.google.com!google-groups.googlegroups.com!not-for-mail
Newsgroups: sci.math
Date: Fri, 21 Jan 2022 10:56:57 -0800 (PST)
In-Reply-To: <sseter$1eiu7$2@solani.org>
Injection-Info: google-groups.googlegroups.com; posting-host=163.182.226.42; posting-account=OWfgwwgAAADQpH2XgMDMe2wuQ7OFPXlE
NNTP-Posting-Host: 163.182.226.42
References: <bb7e1dba-3985-4555-8cf9-5c5421b69769n@googlegroups.com> <sseter$1eiu7$2@solani.org>
User-Agent: G2/1.0
MIME-Version: 1.0
Message-ID: <7141d086-c610-45b2-ba9e-11c4f15048e6n@googlegroups.com>
Subject: Re: DC Proof is the biggest teaching mistake
From: Dan_Chri...@sympatico.ca (Dan Christensen)
Injection-Date: Fri, 21 Jan 2022 18:56:57 +0000
Content-Type: text/plain; charset="UTF-8"
Content-Transfer-Encoding: quoted-printable
Lines: 28
 by: Dan Christensen - Fri, 21 Jan 2022 18:56 UTC

On Friday, January 21, 2022 at 1:19:17 PM UTC-5, Mostowski Collapse wrote:
> Its rather the other way around. You proved too
> much. Please read Landau carefully. He only said:
>
> "To every pair of numbers x, y, we may assign
> exactly one way a natural number"
>
> This can be cast as:
>
> ∀x ∀y ∃!z (x + y = z)
>
> Which translates to:
>
> ∀x ∀y ∃z (x + y = z)
> ∀x ∀y ∀z ∀t (x + y = z & x + y = t => z = t)
>

The same is true of ANY binary function on a given set. You have A=B and A=C, therefore B=C. Nothing profound there. This is certainly no substitute for a formal proof of existence of the addition function on N, as I have given here:

https://www.dcproof.com/ConstructAdditionNew.htm

Dan

Download my DC Proof 2.0 freeware at http://www.dcproof.com
Visit my Math Blog at http://www.dcproof.wordpress.com

Pages:12345678910111213141516
server_pubkey.txt

rocksolid light 0.9.81
clearnet tor